Вы находитесь на странице: 1из 55

Copyright Outline

Chapter 1. Introduction: The Landscape of Copyright


Article I, Sec 8, Cls 8
Congress has the power *t+o promote the Progress of Science and useful Arts, by securing for limited Times to Authors and Inventors the exclusive Right to their respective Writings and Discoveries Copyright/Patent clause Incentives for CREATORS protection for limited time = limited monopoly Securing for a limited time signifies intent to create a public domain (see the Rhetoric of the Public Domain)

The Copyright Office


1) Registers copyrights and issues certificates of registration. 2) Keeps records of registrations, assignments, and licenses of copyright and other matters 3) Regulates deposit of copyrighted materials

Federal Copyright Act of 1790


1) Act effective date: 1790 2) Materials protected: Maps, charts, books 3) Protection begins: When the title of the work is filed for registration w/ stationers company 4) Rights granted: Printing, selling 5) Duration: 14 year original term; 14 year renewal term 6) Notice Required: ?? Donaldson v Beckett Statute of Anne supersedes common law once 14 + 14 expires, work goes into public domain no more common law copyright

Copyright Act of 1909


1) Act effective date: 1905 2) Materials protected: Protected all the writings of an author; did NOT cover unpublished works (except for works not intended for reproduction, such as speeches) a) This caused dual systems of copyright: 1) state protection for unpublished works (i.e. they could be fixated, but not published, and they would not be protected); 2) federal protection for published works

b) After White-Smith (see p. 6), Congress added to the 1909 Act: 1) protection for Mechanical Reproductions of musical compositions (including piano rolls and the then emerging phonograph recordings and 2) compulsory licenses for mechanical reproduction 3) Protection begins: The moment of publication (assuming fixation of a proper copyright notice) 4) Rights granted: 5) Divisibility: Rights were indivisible

Chapter 1. Introduction: The Landscape of Copyright

1/55

6) Duration: First term = 28 yrs; 28-year renewal term 7) Notice Required: Yes proper copyright notice required

Copyright Act of 1976


1) Act effective date: Jan 1, 1978 2) Materials protected: Original works of authorship (see Chapter 2) As of date of enactment, this included 1) literary work; 2) musical works; 3) dramatic works; 4) pantomimes and choreographic works; 5) pictorial, graphic, and sculptural works; 6) motion pictures; and 7) sound recordings 3) Protection begins: The moment of fixation in a tangible medium a) This expanded the scope of works protected federal law compared to the 1909 Act i) ii) More works are fixated then are published. More works fit into federal protection than state protection

b) See 301 (1976 act pre-empts state common-law copyright) 4) Rights granted: 106 enumerated 5 exclusive rights 1) Reproduce; 2) Adapt; 3) Distribute; 4) Perform Publicly; 5) Display Publicly rights are limited by 107-122. 5) Fair Use: 107 codified fair use doctrine see below, p. 45 6) Compulsory/Statutory Licenses: Allow access to copyrighted works if you pay the statutory fee and comply w/ formalities. Original 1976 Act had 4 statutory licenses: Cable Television license ( 111); Mechanical Recording License ( 115); Jukebox license ( 116); Public Broadcasting License ( 118) a) 106 was amended to perform publicly a sound recording by means of a digital audio transmission 7) Computer Programs: 117 established protection for, and the scope of rights in, computer programs 8) Ownership/Divisibility: a) Rights are divisible and transferrable (eg author can assign right to reproduce work to A, and assign the right to adapt the work to B) see 201 and 408(a)

b) Owning a material object w/ copyrighted material doesnt give you the copyright (eg owning a protected CD doesnt mean that you now own the -- see 202) 9) Duration: a) Duration begins at fixation! iii) Individual Authors: Life of Author + 50 years iv) Joint Authors: Life of last living author + 50 years v) c) Anonymous/Pseudonymous & Works for Hire (eg Corporations): The lesser of: a) 75 years from publication, or b) 100 years from creation (fixation?) Individual Authors: Life of Author + 70 years Joint Authors: End of life of last living author + 70 years b) Before CTEA 1998 (see 302(c)):

After CTEA 1998: i) ii)

iii) Anonymous/Pseudonymous & Works for Hire (eg Corporations): The lesser of: a) 95 years from publication, or b) 120 years from creation (fixation?) d) Note: For more on the Sonny Bono Copyright Term Extension Act, see below, p. 25. 10) Termination Rights: Individual authors (and sometimes survivors) have inalienable option to terminate transfers of interest after specified periods of time (see 203, 304(c)) 11) Notice Required: a) Notice required for all published works (at time of enactment of 76 Act) b) could be lost by failure to affix notice (eg 1972 Lou Herard. All Rights Reserved)

Chapter 1. Introduction: The Landscape of Copyright

2/55

12) How to Sue: a) Required Registration of at Copyright Office, as well as Recordation

Protection by Various Acts


1) 1976 Act Covers works created 1/1/1978 and later 2) 1909 Act Covers works created between 1922 and 1/1/1978 3) Public Domain Anything created before 1922 (1/1/1922?) is in the public domain

Berne Convention
4) 1 and most important treaty governing intl copyright relations 5) Berne Convention concluded in 1886; USA joined in 1989 6) Each member nation must afford essentially as strong protection to works originating on other member nations as it does to its own domestic works 7) Works Protected: Literary and artistic works includes every production in the literary, scientific, and artistic domain, whatever may be the mode or form of its expression. 8) Prerequisites to Protection: Member nations must afford automatic protection to copyrightable works originating in other member countries. There are NO FORMALITIES, such as notice or registration 9) Duration of Protection: a) Works of Authors: At least Life of author + 50 years (though member nations may provide for more time) Photographs/Works of Applied Art: At least 25 years
st

b) Anonymous/Pseudonymous Works: At least 50 years from publication c) d) Cinematographic Works: At least 50 years after their making 10) Rights Provided: Member nations must protect the rights of 1) Reproduction, 2) Translation, 3) Adaptation, 4) Public Performance, 5) Public Recitation, 6) Broadcasting, and the 7) Film Rights connection w/ various types of works a) Member nations MUST PROTECT THE MORAL RIGHTS OF Attribution and Integrity b) The Berne Convention does not provide for a right of distribution to the public 11) Protection Based on Country of Origin: c) A works country of origin is generally the place of its first publication i) ii) If the works country of origin is a Berne member, the work is entitled to protection in all member nations If a work is published simultaneously in a Berne Country and a non-Bern Country, the Berne country is the country of origin

iii) If the author is a national of a member nation, the work is protected in Berne countries even if it is first published outside the Berne Union iv) If the work is unpublished and the author is a Berne country national, the work is protected 12) United States Adherence: Moral Rights: The US limited the Copyright Acts express protection of moral rights to visual art other existing federal and state causes of action provide sufficient indirect protection of moral rights in other kinds of works. Notice/Registration: The US eliminated mandatory notice requirements and partially eliminated mandatory registration as a prereq to infringement suits

Digital Millennium Copyright Act


1) Enacted the two treaties adopted by the World Intellectual Property Organization in 1996 a) One on Performances; One on Phonograms

Chapter 1. Introduction: The Landscape of Copyright

3/55

Rhetorics of Protection
Incentives/Utilitarian Conception of Copyright
1) Economic incentive for protection 2) Built on economic incentives a) Reward creators to motivate creators to continued creating b) Capitalism paying the authors 2) Intellectual productions qualify as public goods a) Public goods = producers cannot appropriate their value through sale b) Non-rivalrous = product may be consumed without exhaustion of value -- Non-rivalry means that consumption of the good by one individual does not reduce availability of the good for consumption by others c) Non-excludable = more than one customer who can enjoy the product at one time -- nonexcludability means that no one can be excluded effectively from using the good

d) Motivation to protect pub goods == there is a risk that a suboptimal amount of info will be produced or disseminated. So the law protects these kinds of goods to motivate creators to create them. 3) More of a USA thing

Natural Law Conception of Copyright


1) Author-centric theory 2) John Locke the Labor Model a) a) Property rights theory person is entitled to the fruits of his/her labor Moral rights theory authors should have the right of 1) attribution (credit for making the work), 2) integrity (a third moral right is first publication, but thats not discussed here)
bis

3) Hegel the Personality Model (Moral Rights)

b) See Berne Convention, Article 6 integrity and attribution 4) More of a European thing

REQUIRES that member states protect an authors rights of

The Rhetoric of Misappropriation


1) Motivated by fairness it is not right for one to profit (as a free rider) from the intellectual property of another (not fair to reap where you have not sown) 2) Rooted in quasi-contract and restitution 3) Forms the foundation for right of publicity

The Rhetoric of the Public Domain


1) The law should promote a public domain of material not protected by 2) A purpose of law is to ensure that material should eventually go into the public domain to advance creativity, science, etc

New Economic Rhetoric (Neoclassicist Approach)


1) Like the classic economic approach based on economic considerations 2) Copyright is a mechanism for market facilitation moving existing creative works to their highest socially valued uses by enabling copyright owners to realize the full profit potential for their works in the market

Chapter 1. Introduction: The Landscape of Copyright

4/55

The Rhetoric of Social Dialogue and Democratic Discourse


p 62

The Rhetoric of Deference


1) Judicial Deference 2) cases are fact intensive, so sometimes appellate courts defer to the trial courts

Chapter 2. Prerequisites for Copyright Protection


1. 2. Requires fixation & originality 102(b) denies protection to: ideas, procedures, processes, systems, methods of operation, concepts, principles, or descoveries . . . that are explained, illustrated, or embodied in works otherwise satisfying the laws requirements of originality and fixation

1909 Act vs. 1976 Act


1) Under 1909 Act, protection required ACTUAL PUBLICATION 2) Under 1976 Act, required Creation, i.e. FIXATION on a TANGIBLE MEDIUM

Expression of an Idea
1) 102(b): Protects the EXPRESSION of an idea, but not the idea itself; (C) protects the expression of facts, but not the facts themselves

Fixation (CB 65-78)


1. 101: A work is fixed in a tangible medium of expression when 1) it is embodied in a copy or a phonorecord, 2) it is fixed by or under authority of the author, 3) it is sufficiently permanent or stable to be perceived, reproduced, or otherwise communicated for a period of more than a transitory duration. There are 2 categories of tangible objects in which copyrightable works of authorship can be fixed: a) Copies = Material Objects, other than phonorecords i) ii) on which a work can be fixed by any method now known or later developed, and from which the work may be perceived, reproduced, or otherwise communicated, directly, or with the aid of a machine or device on which sounds, BUT NOT sounds accompanying a motion picture or other audiovisual work, are fixed by any method now known or later developed, and 1) i.e. sounds related to a motion picture would be part of that audiovisual work ii) 3. from which the sounds can be perceived, reproduced, or otherwise communicated, directly or with the aid of a machine or device 2.

b) Phonorecords = material objects (as defined above) (for MUSIC) i)

102(a): To be -able, work must be 1) fixed in a 2) tangible medium of expression that 3) exists now or will be later developed a) Until fixation in a tangible medium occurs, the work may only be protected by state common law Fixation: permanent/stable for more than a transitory duration. NOTE: Vocal oration is not -able

4.

Chapter 2. Prerequisites for Copyright Protection

5/55

5.

Destruction of Fixation: The destruction of the only copy of a fixated work creates problems in proving infringement or depositing a copy of the work with the Copyright Office, but it does not vitiate the copyright. i.e. once a work is fixed, its subsequent destruction does not vitiate its copyright) (see Peter Pan Fabrics v. Rosstex Fabrics) Fixation Not Authorized by Author: To be fixed, a work may be placed in a tangible medium by someone other than the author, but only with the authors authorization. 17 U.S.C. 1101 (Uruguay Round Agreement of GATTthe General Agmt on Tariffs and Trade). (see Martignon below, p. 8) Fixation Simultaneous with Transmission: Normally, an author must fix a new workfor example by writing it down or making a sound or video recording of itbefore publicly performing it in order for the work to be ed at time of performance. a) Live performances being transmitted: 101 provides that fixation simultaneous with performance will be able if the performance is being transmitted eg live sports events, live TV broadcasts

6.

7.

b) Unfixed Works: Fed law will not prohibit members of the audience from copying it (State common law may provide protection) c) 8. Copying the live performance of a musical work: May lead to federal liability pursuant to the neighboring right that performers enjoy in their live musical performances.

Interactive Computer Systems/Video Games: Though interactive, video games ARE fixated for the purpose of copyright protection. (see Midway v. Artic, below p. 6) a) The program that runs the game is fixed in a tangible object b) The audiovisual patterns themselveswhat the player seesare largely repetitive

9.

Common-law copyright protection for unfixed works: (See 1909 Act above, p. 1, for state common law under 1909 Act). Under 1976 Act, 301 allows states to continue providing common-law copyright protection for works that are not fixed in tangible form. But there is little case law. Some cases indicate that protection may be afforded for a persons extemporaneous oral statements, but only if the speaker makes it clear that he intends to claim a proprietary interest in his words. (see Hemingway and Falwell p. 7) c) A copy must be humanly intelligible

10. White-Smith Music Publishing v. Apollo Co (U.S. 1908) d) Facts: Smith copyrighted 2 musical compositions, published as sheet music. Apollo manufactured a player piano that used piano rolls (not sheet music) to reproduce the melodies of the copyrighted pieces. White-Smith sued e) f) Issue: Under the Copyright Act, is a perforated piano roll a copy of the music it reproduces? Holding/Rule: No for a copy of a musical composition to exist, there must be a written or printed record in intelligible notation. Musical sounds that reach us through the sense of hearing cannot be considered copies. The piano rolls are not intended to be read by humans as music they are unreadable by humans, and are used with a machine that produces musical tones. Thus, they are not copies. Note: This was BEFORE the 1909 Act was passed

g)

h) Dissent: The piano roll should be considered a copy 11. Midway Manufacturing Co. v. Artic International, Inc. (N.D. Ill. 1982) a) Computer programs are -able as literary works This is in the 1976 Act, but not the 1909 act b) Facts: Midway (P) manufactured and sold video games, including Galaxian and Pac-Man. Artic (D) made 2 devices: 1) a speed-up kit for Galaxian (that sped up gameplay when attached to the game) and 2) a printed circuit board that is used to create a Puckman game. c) Arguments: Artic argued that the audiovisual aspects of Midways games were not fixed in any tangible medium of expression per 102(a), and didnt qualify for (C) protection. The ROMs in the

Chapter 2. Prerequisites for Copyright Protection

6/55

video games do not contain enough memory to store the entire picture that appears on screen at any moment; instead, they contain symbols that are combined at run-time. d) Issue: Do the images generated by the video game ROMs constitute protected material? e) f) Holding/Rule: Yes. The Copyrighted material is recorded in ROMs and may be reproduced with the aid of the microprocessor. Rationale: The fixation requirement does not require that the work be written down or recorded somewhere exactly as it is to be perceived by the human eye. All that is necessary is that the work is capable of being reproduced . . . with the aid of a machine or device. Note: This reverses White-Smith. Holding: The loading of the ptfs ed software into RAM constituted a fixation and qualified as a copy under the Copyright Act for infringement analysis

g) a)

12. Note 4, p. 78 MAI Systems Corp. v. Peak Computer Inc. (9th Cir. 1993)

b) Note: The specific result of MAI was overturned in 1998 by Title III of the DMCA, but the courts reasoning was not rejected by Congress c) Courts have consistently accepted MAIs holding that a work can be electronically fixed in RAM for purposes of Infringement Analysis

13. Note 6, p. 78 The Office now only allows a single registration for display-generating programs: The proprietor must choose whether the program aspects or the audiovisual aspects predominate and file for registration for only one type of work a) The Copyright Office used to allow separate registrations for video games: 1) for display of the game (as an audiovisual work) and 2) for the underlying program as a literary work.

b) After Midway was decided, the Copyright Office reversed its practice of permitting separate registrations for the display of an electronic game as an audiovisual work, and also for the underlying computer program as a literary work. 14. Note 7, p. 79 Copyright exists in works e.g. of sporting events, where the recording is done simultaneously with the live broadcast -- it is fixated as its recording a) 101 says a work consisting of sounds, images, or both, that are being transmitted, is fixed . . . if the fixation is being made simultaneously with its transmission.

15. Note 8, p. 79 Unfixed works and common law copyright b) Estate of Hemingway v. Random House, Inc. (N.Y. 1968) i) Hemingways Estates argued that things that late Hemingway never wrote down (e.g. anecdotes, reminiscences, literary opinions, and revealing comments) were literary property protectable under New York law Holding: No common-law copyright for unwritten utterances.

ii)

iii) Rationale: Hemingway did not indicate an intention to claim the utterances in question as his property. i.e. he did not mark off the unwritten utterances as separate from any other everyday speech. c) Falwell v. Penthouse International, Ltd. (W.D. V. 1981) i) ii) Holding: No common-law copyright for the quotations Rationale: There was no defined segregation, either by design or by implication of any of ptfs expressions of his thoughts and opinions of the subjects discussed, that would aid in identifying ptfs purported copyrighted material

d) Notes: These cases suggest that common-law copyright may exist if the speaker says he intends for the words he says to be copyrighted?? e) See also Kelley v. Chicago Park District

Chapter 2. Prerequisites for Copyright Protection

7/55

Note 12, p. 81 Fixation, International Protection, and Bootlegging a) Note: Many countries protect fixed AND unfixed works w/o differentiation. Neither the Berne Convention (which the US joined on Mar 1, 1989), nor the Universal Copyright Convention (which the US has been part of since 1955) limits members to protecting only fixed works.

b) 1101 gives performers a civil cause of action for unauthorized fixation of, or trafficking in, live musical performances. (Note: 1101 is outside the Copyright Act, which is housed in chapters 1-8 of Title 17). c) Criminal penalties under 18 U.S.C. 2319A supplement the civil cause of action

d) United States v. Moghadam (11th Cir. 1999) i) ii) Holding: 2319A is constitutional Rationale: The Copyright Clause does not envision that Congress is forbidden from extending copyright-like protection under other constitutional clauses, e.g. the Commerce Clause

United States v. Martignon (2d Cir. 2007) i) ii) Holding: 2319A is constitutional and may be upheld under the Commerce Clause Rationale: The section is only a criminal prohibition. Tt does not secure an exclusive right within the meaning of the clause, because it does not create property rights upon authors or inventors, or allocate those rights among claimants to them.

Originality (CB 82-127)


1. 2. Originality is not novelty or the quality of being new. Originality is required by the Constitution (Art I, Sec 8, Cls 8) a) The Trade-Mark Cases: Writings for a work to be classified as a writing of authors (under Art I, 8, Cls 8), originality is required.

Burrow-Giles: Author = he to whom anything owes its origin; originator; maker 3. Two PRIMARY FACTORS of originality: a) Independent Creation By Author i.e. not copied from someone elses work

b) Modest Quantum of Creativity i.e. this is a low standard c) NOTE: The Sweat of the Brow (I worked hard to create something) doctrine DOES NOT invoke protection

Originality vs Novelty
4. 5. Alfred Bell & Co v. Catalda Fine Arts, Inc (U.S. 1884) a) Original means that a work owes its origin to the author. No large measure of novelty is necessary Feist Publications, Inc v. Rural Telephone Service Co (U.S. 1991) b) Sweat of the Brow IS NOT A SUBSTITUTE FOR ORIGINALITY c) Facts, laws of nature, etc are NOT copyrightable; BUT, authors original selection & arrangement CAN BE copyrightable

d) Note: For the long brief, see Compilations on p. 17.

Chapter 2. Prerequisites for Copyright Protection

8/55

Authors and Their Writings


1. Burrow-Giles Lithographic Co v. Sarony (U.S. 1884) a) e) f) g) Facts: Burrow-Giles (D) sold copies of a posed photo of Oscar Wilde on which the original photographer, Sarony (P), had a . P sued and won. Burrow-Giles (D) appealed. Arguments: Burrow-Giles (D) argued that s cannot be granted for photos because they are not writings, but just duplications of representations Issue: Is a photograph a writing for purpose, or just a duplication of a representation of space? Holding: A photograph is a writing for the purpose of Copyright infringement analysis

h) Rationale: A photograph such as this contains creativity. It is an embodiment of the authors creative expressionselection of costumes, pose, lighting, etc. The photographer is then the author, the photograph owes its origin to the author, and it is a writing that deserves . 2. Note 6, p. 95 Classifying the elements of originality? a) Mannion v. Coors Brewing Co (S.D.N.Y. 2006) i) ii) Photographs are capable of originality inn three (non-mutually exclusive) respects: Rendition e.g. angle of shot, light and shade, exposure, and effects achieved by means of filters or developing techniques

iii) Timing e.g. the photographers decision to seize a moment in time and reduce it to an image (e.g. being at the right place at the right time) but seriously, how much of that is just luck? Thats not creative iv) Creation of Subject Creating the scene or subject to be photographed; e.g. posing the subject in a particular setting 3. Bleistein v. Donaldson Lithographing Co (U.S. 1903) b) Supreme Court did away with distinction between fine art and non-fine art as long as there is some minimal creativity & originality, then its able c) Facts: In order to advertise his circus, Wallace hired Bleistein (P), who produced three chromolithographs depicting some of the acts in the show, and Wallace in the corner. Donaldson Lithographic (D) produced reduced copies of the chromolithographs, and P sued to recover penalties for infringement.

d) Arguments: D argued that chromolithographs do not meet the criteria for protection in lieu of a provision in the act of 1874 that the words engraving, cut, and print shall apply only to pictorial illustrations or works connected with the fine arts. e) f) g) Procedure: D won directed verdict, which was upheld on appeal. Bleistein appealed further. Issue: Are chromolithographs, designed for advertising purposes, barred from copyright protection? Holding: No. The chromolithographs (posters) are able. The fact that a chromolithograph was produced as an advertisement does not change the fact that such pictorial illustrations are copyrightable.

h) Rationale: The chromolithograph is a pictorial illustration that may be copyrighted. Also, copyright does not distinguish between fine art and non-fine art. Any work may be copyrighted if it meets the statutory criteria. i) j) Note: This was before the Act of 1909 Dissent: A pictorial illustration must have some intrinsic value other than its function as an advertisement to be copyrightable, as copyright protection is not offered to mere labels simply designating or describing an article. A connection with the fine arts is what gives this needed intrinsic value, but here, there is only advertising value.

Chapter 2. Prerequisites for Copyright Protection

9/55

Originality Revisited
1. Meshwerks, Inc. v. Toyota Motor Sales U.S.A., Inc. (10 Cir 2008) k) Facts: Saatchi (advertising agency) and Toyota began working on ad campaign involving digital models of Toyota cars. Saatchi and Toyota hired Grace & Wild, Inc (G&W) to supply digital models. G&W subcontracted Meshwerks to do digitization and modeling. G&W then refined the initial models and sent to Saatchi for advertising. Arguments: Meshwerks argued that it contracted w/ G&W for only one use of the models (in one TV commercial), not other ads. Thus, Saatchi/Toyota committed infringement by reusing/redistributing the models that Meshwerks created.
th

l)

m) Issue: Did Toyotas use of the models in more types of ads than Meshwerks contracted for (or intended?) constitute infringement? n) Holding: No Toyota did not infringe o) Rationale: 1) Meshwerks models were just unadorned duplications of Toyota cars, without any individualizing features (i.e. no paint, no re-touching, no scene selection, etc.). Meshwerks did not add any expression The putative creator who merely shifts the medium in which anothers creation is expressed has not necessarily added anything beyond the expression contained in the original. (says Nimmer on Copyrights) 2) Meshwerks intent was to be unoriginal. The idea was to copy Toyotas vehicles exactly as they appear, without adding any expression. p) Notes: Compare to Burrow-Giles

Idea vs Expression
2. Baker v. Selden (U.S. 1880) a) There is no copyright protection for ideas b) Facts: Selden (P) copyrighted a book in which he used an introductory essay explaining his system of bookkeeping followed by blank forms to put the system to use. He had arranged the columns and headings so that the entire operations of a day, week, or month was on a single page or on two pages facing each other. Baker (D) started selling forms w/ differently arranged columns and headings to achieve the same result. c) Procedure: Selden (P) successfully brought suit for infringement. Baker (D) appealed. Court reversed Issue: Does a copyright on a book explaining an art or system preclude others from using the system or the forms incidental to such use? Holding: No. A copyright on a book explaining an art or system protects only the authors unique explanation and does not preclude others from using the system or the forms incidental to such use. Rule: When copying is the ONLY WAY to implement the idea underlying the expression, copying will not constitute copyright infringement

d) Arguments: D argued that the forms were not copyrightable e) f) g)

h) Rationale: To find that a protected against the use of the system itself or the forms necessary to such use would be to grant patent-type protection without requiring a show of novelty (which is required for patent protection). Copyright is based on originality, not novelty. Therefore, Copyright protects the explanation, and not the use of the system explained. Thus, Selden (P)s copyright could not give the exclusive right to use the bookkeeping system or the forms necessary to such use. i) See also: Miller v Universal City Studios p. 11. (Ideas cannot be copyrighted; only expressions can be.)

Chapter 2. Prerequisites for Copyright Protection

10/55

The Merger Doctrine


1. 2. Merger Doctrine: When there is only one or a limited number of ways to express an idea, the expression merges with the idea and becomes unprotectible Morrissey v. Procter & Gamble (1st Cir. 1967) a) Facts: The ptf, Morrissey, is the owner of a set of rules for a sales promotional sweepstakes-type contest involving the social security numbers of the participants. The substance material was simple, and the def even admitted that there was more than one way of expressing even the simple substance. Rule: When the uncopyrightable subject matter is very narrow, so that the topic necessarily requires only a limited number of forms of expression (if not just one), to permit copyrighting could exhaust all possibilities of future use of the substance.

b) Holding: Procter & Gamble win, even though they essentially wholesale-copied the ptfs substance. c)

d) Rationale: The rules documented were so simple that it was easier to hold that Copyright does not extend to the subject matter at all. this is the merger doctrine at work. Court relied on Baker e) Note: See also Computer Associates v. Altai (on p. 43). The underlying principle of the merger doctrine is: When there is essentially only one way to express an idea, the idea and its expression are inseparable, and copyright is no bar to copying that expression. With computers, this means that when specific instructions are the only and essential means of accomplishing a given task, their later use by another will not be infringement (even if they are copyrighted).

Thin Copyright
1. Thin Copyright: Instead of finding a merger when the number of ways of expressing an idea is limited, some courts prefer to recognize copyright, but refuse to find infringement unless the defs copying is nearly exact

Government Works
1. 105: Govt works are not copyrighted a) protection under this title (17 U.S.C. the Act) is not available for any work of the US Govt b) BUT US govt MAY receive and hold s transferred to it by assignment, bequest, or otherwise

Other preliminary considerations (CB 132)

Chapter 3. Works of Authorship


Original Works of Authorship Under 102
102(a) lists 8 categories of works protected by copyright: 1) literary works; 2) musical works, including any accompanying words; 3) dramatic works, including any accompanying music; 4) pantomimes and choreographic works; 5) pictorial, graphic, and sculptural works; 6) motion pictures and other audiovisual works; 7) sound recordings; and 8) architectural works

Literary Works, Including Computer Software


Miller v. Universal City Studios, Inc (5 Cir 1981)
th

Chapter 3. Works of Authorship

11/55

a)

Copyright protection extends only to an authors expression of facts, and not to the facts themselves (see Burrow Giles; Baker v Selden; Feist)

b) Facts: Miller (P) wrote a book about an unsuccessful kidnapping that occurred during the 1970s. Universal City Studios, Inc. D approached P about purchasing the rights to the book, but negotiations never reached a final agreement. Nonetheless, D produced a TV-movie about the event. c) Procedure: P sued D and the screenwriter for copyright infringement. The district court instructed the jury that research is copyrightable. The jury awarded 200k to P. D appealed. Holding: No. The product of research cannot be copyrighted. Ideas and facts are not copyrightable; only the expression of them is copyrightable Rationale: Copyright law embodies the notion that facts are in the public domain. Once published, a fact can be used by anyone. The district courts instruction essentially told the jury that facts can be copyrighted. This is contrary to the law

d) Issue: Can the product of research be copyrighted? e) f)

Apple Computer, Inc. v. Franklin Computer Corp (3d Cir. 1983) Computer software is copyrightable as a literary work g) Facts: Apple (P) made an operating system. Franklin (D) sells computer (ACE 100) designed to be Apple Compatible so peripheral equipment and software for the Apple II could be used with the ACE 100. Apple sued because Franklin copied Apples operating system computer programs to achieve the compatibility.

h) Argument: On appeal, the central contention by D was that computer operating system programs, as distinguished from application programs, are per se uncopyrightable because they are either a process, system, or method of operation. i) Procedure: The district court denied Apples (P) motion to preliminarily enjoin Franklin (D) from infringing the s that Apple held on 14 computer programs. The dist ct was persuaded by Ds argument that the OS program is part of a machine, and not copyrightable. Issue: Are computer operating system programs per se uncopyrightable? Holding: No. After the 1980 amendment to the Copyright Act, it is firmly established that computer programs are copyrightable. Rationale: The Copyright Act defines a computer program as sets of statements/instructions used in a computer to bring about a result. There is nothing to suggest that OPERATING SYSTEM programs, as distinguished from APPLICATION programs, are uncopyrightable. Apple did not seek to copyright the method by which the computer is instructed to perform its functions. It sought only to copyright the instructions themselves. i) Does protect user interface? probably not imagine if you could the user interface, and therefore, every computer program had to be different. There would be no standards (e.g. F4 key to exit) Lotus Development Corp v. Borland International, Inc. (U.S. 1995) also discussed in Chapter 8 after Altai (notes CB p. 694) 1) Essentially, a user interface is NOT copyrightable it is not copyrightable expression. 2) Its more of a method of operation--the means by which a person operates something, whether it be a car, a food processor or a computernot copyrightable 3) Facts: The underlying code for Lotus 1-2-3 and Borlands Quattro Pro spreadsheet were different, but Borland copied Lotuss menu command hierarchy 4) Lotuss menu system for Lotus 1-2-3 (spreadsheet) was not a n) Note 7, p. 164 Reverse engineering

j) k) l)

m) Note 6, p. 163 Copyrighting User Interfaces (this appears in Chapter 8, also)

ii)

Chapter 3. Works of Authorship

12/55

i) ii)

Though reverse engineering usually involves copying, courts have carved out a substantial exception for reverse engineering under fair use See e.g. Sega Enterprises Ltd v. Accolade, Inc (9th Cir. 1992) in Chapter 10

Musical Works
a) Musical works include both the instrumental component of the work and any accompanying words b) Musical works have 2 s i) Sound Recording arise out of the joint authorship of those who participate in the recording process

Underlying composition the melody, rhythm, etc. c) Fixation may be written on paper, or electronically recorded on CD, disk drive, etc i) Note: A phonorecord may simultaneously fix musical works, literary works, and sound recordings

d) Originality may be satisfied through melody, harmony, or rhythm (individually, or in combo w/ each other) e) Taking lyrics alone constitutes infringement f) Musical works can be incorporated into other, larger composite works, and maintain -ability i) ii) iii) The 1909 Act prohibited separate registration of musical works that were first published as part of a motion picture The 1976 Act removed that bar Arrangements of musical works are protected under 103(a) as derivative works

One of the most valuable exclusive rights of owner is to prepare and distribute mechanical reproductions 1976 Act refers to these as phonorecords -- 106 BUT under 115 Mechanical License, the owner is allowed first right to authorize the creation and distribution of phonorecords. BUT after that, anyone who follows the procedures in the statute and pays the statutory fees can cove the original recording w/o the express permission of the owner, as long as the new version does not change the basic melody or fundamental character of the work.

Dramatic Works
a) Dramatic works illustrate how the various categories in 102(a) may overlap i) ii) iii) i) Music accompanying a dramatic work may be copyrighted separately rather than under the dramatic works category The script for a dramatic work may be protected as a literary work A film dramatization of a screenplay is protected as a motion picture 110(2) exempts from liability for infringement certain performances by nonprofit or governmental entities, ONLY IF THE WORK IS A NONDRAMATIC LITERARY OR MUSICAL WORK (or a reasonable and limited portion of any other type of work Performing Rights Societies (ASCAP, BMI) and Licensing Organization (e.g. Harry Fox Agency) limit their activities to nondramatic musical works

b) But the dramatic works category is important

ii)

Pantomimes and Choreographic Works


a) Fixation usually occurs by videotaping the performance of the pantomime or choreography i) Choreography may also be written in notation, using Laban or Benesh systems

Chapter 3. Works of Authorship

13/55

ii)

To be able, the work must be described in enough detail to enable the work to be performed from that description Choreography 1. 2. What IS PROTECTED: Hard to determine what is original; but original routines and original arrangements of pre-existing routings are able What is NOT PROTECTED: FOLK or SOCIAL steps are NOT PROTECTIBLE (i.e. commonplace steps like the foxtrot are not able WHAT IS PROTECTED: Originality is found in the performers art in imitating or acting out situations, characters, or some other events with gestures and body movement WHAT IS NOT PROTECTED: Stock movements and styles

b) Work must be original i)

ii)

Pantomime 1. 2.

Pictorial, Graphic, and Sculptural Works


a) 102(a)(5) provides for (C) of PGS works b) Includes two-dimensional and three-dimensional works of fine, graphic, and applied art; photographs, prints and art reproductions; maps, globes, charts, diagrams, models, and technical drawings c) Usu the issue is with Originality i) ii) iii) e.g. Maps maps are generally rearrangements or depictions of factual matter The courts USED TO enforce the sweat of the brow theory w/ maps, but they STOPPED, even before Feist United States v. Hamilton (9th Cir. 1978) (note case p. 174) 1. 2. Argument: Def argued that the map he copied was not subject to because it was simply a synthesis of info already in the public domain Holding: Maps are only required to display something originalarrangements and combinations of facts are as long as they are not merely trivial variations of info w/in the public domain. The elements of authorship: In this case were Selection, Design, and Synthesis; they satisfied the originality standard

3.

Mazer v. Stein (U.S. 1954) Utilitatarian vs. Artistic Aspects (Utilitarian aspects v. artistic aspects) a) Facts: Plaintiff/Respondent Stein (P) made a designed statuettes and obtained a . Defendant/Appellants Mazer (D) designed a lamp using the statuette as the base and started mass producing the lamps.

b) Procedure: P sued for infringement, seeking damages and injunctive relief. Dist ct found infringement and awarded damages and enjoined further infringement. D appealed. Appellate ct affirmed dist ct. Supreme Ct granted cert c) e) f) Argument: D argued that a practically useful article could be protected only by a patent, not by a copyright. Holding: Yes. The patentability of the statuettes, whether fitted as lamps or not, does not bar copyright as works of art. Rationale: Statutory construction: Neither the patent law nor the law contain any provisions excluding an item from falling within both areas. The fact that an item may be patentable does not preclude it from being ed. Allowing a patentable item to be copyrighted would not discourage invention or innovation. The court analyzed law from 1870, 1874, 1909, and 1976 Concurrence: Douglas concurs, but the Constitution only gives Congress the power to grant s on writings. Therefore, the copyright law may exceed Congress constitutional authority. The case should be submitted for reargument and rebriefing on this issue d) Issue: Can an article having a utilitarian application be copyrighted?

g)

Chapter 3. Works of Authorship

14/55

h) Notes: The court in Mazer employed test #3 listed in Pivot Point Pivot Point International, Inc. v. Charlene Products, Inc. (7th Cir 2004) Useful Articles Separability of Artistic and Utilitarian Aspects of a Work (Useful articles discussion) a) Facts: Pivot Point and Charlene are two companies involved in the makeup/hair design industries (i.e. not makeup manufacture, but design the art of makeup and hair design). Pivot Point hired an artist (Heerlein) to design Mara (a mannequin). Heerlein did so and assigned the rights to Pivot Point. Pivot Point sued Charlene for copying the design of a mannequin head used for hair design instruction/practice/competition. Charlene claimed the head was a "useful article" that was not copyrightable. Pivot Point claimed the head was not a useful article; that the design was a work of art that is copyrightable (its inherent nature is to portray the "hungry look" appearance of a runway model)

b) Issue: Is the mannequin copyrightable? Is it a "useful article?" Is the design "separable" from the utilitarian aspects of the article (i.e. the head)? c) Holding: Head IS copyrightable (Reverse Dist Ct ruling) (Ct affirms that the head is a useful article) d) Rule: 1) If an article is not useful as the term is defined in 101, then it is a PGS work entitled to copyright protection (assuming the other requirements of the statute are met). 2) The design of a useful article shall be considered a PGS work only if, and only to the extant that, such design incorporates PGS features that CAN BE IDENTIFIED SEPARATELY FROM AND ARE CAPABLE OF EXISTING INDEPENDTLY OF, the utilitarian aspects of the article e) Rationale: Conceptual separability exists when the artistic aspects of an article can be conceptualized as existing independently of their utilitarian function. This independence is informed by whether the design elements can be identified as reflecting the designer's artistic judgment exercised independently of functional influences. Passage (the guy who hired Heerlein) did not give Heerlein any design requirements if he had, that would weigh against a finding that the mannequin was mainly an artistic endeavor. By contrast, Heerlein had free reign to design the mannequin Dissent: The Mara (mannequin/makeup head) has only functional attributes. Thus any physical separation of a portion of her would not be independent of her utilitarian aspects. She is sold as a teaching device. There is nothing in Mara that we could physically remove that would not be part of Maras utility as a teaching aid Notes: Pivot Point cites several tests for determining when the artistic and utilitarian aspects of useful articles are conceptually separable (this is taken from lecture notes) i) ii) iii) Kieselstein-Cord v. Accessories by Pearl, Inc (2d Cir 1980) The article features are primary and the utilitarian features are subsidiary Nimmer on Copyright The useful article would still be marketable to some significant segment of the community simply because of its aesthetic qualities Carol Barhhart Inc. v. Economy Cover Corp 2d Cir. 1985) The article stimulates in the mind of the beholder a concept that is separate from the concept evoked by its utilitarian function (Charlene supports this test) Brandir Intl Inc. v. Cascade Pac. Lumber Co. (2d Cir. 1987) The artistic design was not significantly influenced by functional considerations Goldstein on Copyright The artistic features can stand alone as a work of art traditionally conceived, and the useful article in which it is embodied would be equally useful without it (Pivot Point supports this) Patry Copyright Law & Practice The artistic features are not utilitarian THE COURT BLENDS KIESELSTEIN and BARNHART (see p. 186)

f)

g)

iv) v)

vi) vii)

Sound Recordings
a) A sound recording IS NOT THE SAME AS THE MUSICAL WORK IT CAPTURES

Chapter 3. Works of Authorship

15/55

i) ii)

The SOUND RECORDING is the recording itselfthe rendition in the studio, as recorded that day The MUSICAL WORK is the melody, harmony, and lyrics

b) 102(a)(7) protects the particular aggregate of sounds collected in the (C) work called the SOUND RECORDING, and NOT THE MUSICAL WORK c) Requires originality and fixation in a tangible medium

Architectural Works
a) 101 Architectural works protect the design of a bldg i) Design includes 1) overall form and 2) arrangement and composition of spaces and elements in the design Creativity in architecture frequently appears as selection, coordination, or arrangement of unprotectible elements into an original, protectable whole An architect may incorporate new, protectable design elements into otherwise standard, unprotectible bldg features Interior architecture may be protected NOTE: Architectural works do NOT qualify as Pictorial, Graphic, or Sculptural works under 102(a)(5)

b) 102(a)(8) Protects architectural works i) ii) iii) iv) c)

Specifies material objects that may embody an architectural work: A building, architectural plans, or drawings. i) ii) iii) The work should be examined to determine whether there are original design elements present, including overall shape and interior architecture If such elements are present, examine whether the design elements are functionally required If desn elements are NOT functionally required, the work is protectable w/o regard to physical or conceptual separability

d) Two-step analysis envisioned for infringement of Architectural Works

Shine v. Childs (SDNY 2005) Total Concept & Feel test Facts/Procedure: Ptf Shine (an architecture student) developed two models for a skyscraper design, a preliminary model, entitled Shine 99, and a more sophisticated model, entitled Olympic Tower. The Olympic Tower consisted of a twisting tower with a symmetrical diagonal column grid, expressed on the exterior of the building, that follows the twisting surface created by the floor plates geometry. Shine presented his design to a jury panel of industry luminaries. Among the members of the panel was the defendant Childs. Childs was selected as design architect and project manager for a bldg at the new World Trade Center site. Childs exhibited two scale-models of the Freedom Tower, various images, and a slide show explaining the underlying design principles. The Freedom Tower tapers as it rises and has two straight, parallel, roughly triangular facades on opposite sides, with two twisting facades joining them. Shine believed the design for the Freedom Tower looked identical to his earlier designs for Shine 99 and the Olympic Tower. Shortly thereafter, Shine registered his models with the U.S. Copyright Office and sued Childs and his firm, SOM, for copyright infringement Rule: To establish infringement, a plaintiff must show that (1) his work was actually copied, and that (2) such copying constituted an improper appropriation. Because direct evidence of actual copying is difficult to obtain, actual copying can be established through proof of access to the plaintiffs work and probative similarity between the plaintiffs work and the allegedly infringing work. Analysis: Childs had access to Shines designs. The court used the total concept and feel test, traditionally applied to aesthetic arts, relies on the visceral response of the ordinary observer to a work as a whole. The court rejected the analytic dissection approach, originally developed for complex, functional works such as computer software, filters out the unprotectible elements of a plaintiffs work prior to comparison. Total Concept & Feel Any lay observer examining the two towers side by side would notice that: (1) each tower has a form that tapers and twists as it rises, (2) each tower has an undulating, textured diamond shaped pattern covering its facade, and (3) the facades diamond pattern continues to and concludes at the foot of each tower,

Chapter 3. Works of Authorship

16/55

where one or more half diamond shapes open up and allow for entry. These combinations of these elements gives the two towers a similar total concept and feel that is immediately apparent even to an untrained judicial eye. Holding: Shine wins. Freedom Tower infringed on Olympic Tower Copyright Law distinguishes btwn the work of authorship itself and the material object in/on which the work is fixed. (see 102. Also, refer to Chapter 4)

Derivative Works and Compilations Under 103


Derivative Works
a) 103 protects compilations and derivative works b) 102 works are first-generation works (e.g. made by the author); 103 works are second generation (e.g. works based on already extant material) c) 2 types of derivative work: i) ii) Public-Domain original No licensing concerns Protected Original Must get rights from holder 1. Derivative author gets ONLY IN HIS OWN ORIGINAL MODIFICATIONS TO THE ed ORIGINAL WORK

d) 5 formulations Standards for evaluating Derivative Works i) ii) iii) iv) v) Trivial variation from original work Minimal variation Distinguishable variation Alfred Bell case Substantial variation L Batlin case, Gracen case Sufficiently gross variation Derivatives
th

Schrock v. Learning Curve International, Inc. (7 Cir 2009) Confirms that derivative works

Compilations
a) c) Originality is a constitutional requirement Facts are not copyrightable (facts do not owe their origin to an act of authorship) i) e.g. scientific, historical, biographical, news of the day are not coyprightable they are in the public domain Compilation author generally chooses which facts to include, in what order to place them, and how to arrange the collected data b) The sine qua non of copyright is originality

d) Compilations generally are copyrightable (they may possess the requisite originality) i)

Feist Publications, Inc. v. Rural Telephone Service Company, Inc. (U.S. 1991) e) f) Did away with sweat of the brow theory for protections Facts: Rural = certified public utility that provides phone service to several communities in NW Kansas. As part of phone svc monopoly, Rural publishes an annually updated phone directory white pages alphabetically list names, towns & phone #s of subscribers; yellow pages list Rural's biz subscribers alphabetically by category. Rural collects its own phone subscriber data. Feist = publishing co' specializing in area-wide phone dirs (covers larger range than Rural). Feist & Rural compete w/ each other for Yellow Pages advertising. Feist isn't a phone co it pays for right to use phone listings from regional phone carriers. Only Rural refused to license its listings to Feist; Feist used white pages listings w/o Rural's consent. 1309 of Feist's listings in its 1983 directory were identical to Rural's, including 4 fictitious listings that Rural had inserted to detect copying.

Chapter 3. Works of Authorship

17/55

g)

Procedure: Rural sued for infringement in Dist Ct for D. Kan. Dist Ct granted sum judg to Rural. Tenth Circuit affirmed. Supreme Court granted cert to determine whether the copyright in Rurals directory protects the names, towns, and telephone numbers copied by Feist

h) Issue: To be copyrightable, must a work be original to the author and possess at least some minimal degree of creativity? i) j) Holding: Yes. To be a copyrightable, a work must be original to the author and possess at least some minimal degree of creativity Rationale: While facts are not copyrightable, compilations of facts are. Originality, not the "sweat of the brow," is the touchstone of copyright protection in directories and other fact-based works. Copyright protection extends only to the components of a work that are original to the author. Feist, by taking 1309 names, towns & phone #s from Rural's white pages, did NOT copy anything that was "original" to Rural the names, phone numbers, etc are facts. Also, Rural did not select, coordinate, or arrange these uncopyrightable facts in an original way. Alphabetical order is not creative. Notes: To establish infringement, Ptf must show (1) ownership of valid copyright, and (2) copying of constituent elements of the work that are original. Feist conceded point 1, but Rural could not prove point 2. Note 5, p. 251 Best-of Lists Selection i) Silverstein v. Penguin Putnam, Inc. (2d Cir. 2004) 1. Facts: Ptf collected unpublished poems of Dorothy Parker. Ptf tried to have PP publish; PP passed, but kept a copy. Later, PP published the poems of Dorothy Parker, published & unpublished (e.g. all of the Ptfs selections are included in another compilation, but with a presumably different arrangement). Ptf sued. Holding: Ptf loses his selection of poems were obvious ones that required no creative thought.

k) l)

2.

m) Note 8, p. 252 What is Original Arrangement? i) Assessment Technologies of Wisconsin, LLC v. WIREdata, Inc (7th Cir. 2003) 1. Facts: Ptf created a computer program to facilitate the collection & storage of real estate data by municipal tax assessors. The prog sorted data into 456 fields grouped in 34 master categories Holding: The court found the arrangement to be copyrightable; but the underlying data was factual information and not copyrightable. Ptf could not prevent municipalities from extracting and copying the data for the defendant, as long as the def sorted the data in a different manner.

2.

n) Note 9, p. 252 Soft and Hard Facts and the Merger Doctrine i) CCC Information Services v. Maclean Hunter Market Reports, Inc. (2d Cir. 1994) 1. 2. 3. Facts: Ptf published the Red Book of estimated used car values, based on the professional judgment of its editors, who consulted a variety of sources. Holding: The values were able. The values were neither reports of historical prices, nor mechanical derivations of such prices. Court distinguished between building block ideas (which are subject to the merger doctrine because they must be freed up to assist the understanding of future thinkers), and approximate statements of opinion (which are infused with the authors taste or opinion and need not be subject to the merger doctrine) Holding: Court held that NYMEXs daily settlement prices for futures contracts were not copyrightable, based on the merger doctrine

ii)

BUT SEE New York Mercantile Exchange, Inc. v. IntercontinentalExchange, Inc (2d Cir. 2007) 1.

o) Note 11, p. 253 More on Selection, Arrangement, Coordination, and Copyrightable Elements i) West Publishing Co. v. Mead Data Central, Inc. (8th Cir. 1986)

Chapter 3. Works of Authorship

18/55

1.

Facts: West was the premier printed source of legal authority. Lexis (MDC) used Wests star paging as-is in its online docs. West sued MDC for stealing selection/arrangement/coordination by taking page #s Holding: Eigth Cir affirmed Dist Ct ruling properly enjoining MDs planned star pagination to Wests hardbound books. Though the laws/cases are non-copyrightable facts, Wests hard volumes were copyrightable because selection, arrangement, and other elements were original works of authorship.

2.

p) Note 15, p. 255 BUT SEE Hyperlaw this conflicts with West v. MDC i) ii) Matthew Bender & Co & HyperLaw, Inc. v. West Publishing Co (2d Cir. 1998) Facts: West made several modifications to non-copyrightable law case material, including: 1) inserting standardized case captions, court info, and date info; 2) including the names of attorneys involved; 3) providing references to subsequent developments in reported decisions; 4) supplying parallel or alternative citations to authorities relied upon in those decisions. None of this data was routinely included in the court reports that West collected to make its reports. HyperLaw wanted to include all of Wests modifications in its CD-ROM based products. Holding: Relying on Feist, Wests editorial activities were lacking in originality, creativity, or both; especially since the number of options for how to represent the data were limited.

iii)

Database Protection
Notes (CB 257-59)

Chapter 4. Ownership and Transfers


Initial Ownership
Scenarios for initial ownership 1) Sole author working alone author owns 13) Author working with amanuensis (eg secretary, scribe) author owns , not the amanuensis 14) Joint author (eg Author 1 working with Author 2) collaboratively all joint authors own as tenants in common (assuming they meet the requirements for JA see Joint Authorship at p. 20) Joint Authors Rights a) Any jt auth can give a non-exclusive license to someone else to exploit the work w/o permission of co-author Cannot give exclusive license w/o co-authors permission

b) Any auth can exploit the work w/o co-auth permission, but MUST SHARE ROYALTIES c) 15) Employer employs employee as independent contractor IC owns 16) Employer directs employee to create work w/in the employees scope of employment Employer owns 17) Work made for hire, specially commissioned work Employer owns Note: Scenarios 5 & 6 are Works made for hire under 101

Works Made for Hire


101 Work Made For Hire (p. 14 supp) is: 1. A work prepared by an employee within the scope of his/her employment, OR

Chapter 4. Ownership and Transfers

19/55

2.

A work specially ordered or commissioned for use as 1) a contribution to a collective work; 2) part of a motion picture or other audiovisual work; 3) a translation; 4) a supplementary work; 5) a compilation; 6) an instructional text; 7) a test or answer material for a test; 7) an atlas Problem: 101 does not define employee or scope of employment RULE: For a work to BE a work made for hire: The parties must expressly agree in a written instrument signed by them that the work shall be considered a work made for hire Termination of Transfer IS NOT available for the author of a work made for hire (because the independent contractor is not the author)

3. 4. 5.

Community for Creative Non-Violence v. Reid (U.S. 1989) Established the test for determining works made for hire 1) Facts: CCNV + Snyder hired Reid to sculpt a nativity scene. Snyder would sculpt 3 people, CCNV would provide a steam vent and pedestal for special effects. The parties did not sign a written agreement; neither mentioned copyright. Reid expected 2 things: 1) He would own and 2) CCNV would own the statue. 2) Procedure: Lower ct found that CCNV = owner. Ct of appeals found for Reid. S. Ct. granted cert; affirmed for Reid. 3) Snyder + CCNV sued Reid for return of the sculpture and a determination of copyright ownership. Dist Ct granted a preliminary injunction ordering Reid to return the sculpture. Dist Ct ruled that the sculpture was a "work made for hire" under 101 of the Copyright Act, and that Snyder, as trustee for CCNV, was the exclusive copyright owner. Ct of Appeals reversed and remanded, holding that Reid owned the copyright because the sculpture was NOT a work for hire. U.S. S.Ct. granted certiorari and affirmed the Ct of Appeals. i.e. Reid wins 4) Issue: Is the sculpture a work prepared by an employee within the scope of his or her employment under 101(1)? 5) Holding: Reid = owner. The court uses the Common Law of Agency to determine what an employee is 6) Rule: 4 Tests for determining a work made by an employee: 1) The hiring party retains the right to control the end product; 2) The hiring party has wielded actual control wrt the creation of the end product; 3) The term employee carries is common law agency meaning; 4) An employee is a formal salaried employee. Factors relevant in determining employment under agency None of these factors is determinative; they must be balanced: 1) Skill required by hired party (higher required skill weighs in favor of actual creator); 2) source of tools/instruments; 3) Location where actual work was done; 4) Length of relationship between parties; 5) Whether hiring party has right to assign addl projects to the hired party; 6) The hired partys role in determining how long to work; 7) The method of payment; 8) The hired partys role in hiring and paying assistants; whether the work is part of the regular business of the hiring party; 9) whether the hiring party is in business; 10) the provision of employee benefits; 11) the tax treatment of the hired party 7) Rationale: Reid was an independent contractor. Reid is a sculptor (skilled occupation. He supplied his own tools. He worked in his own studio. He was retained for less than 2 months (a short time). CCNV had no right to assign additional projects while he was making the sculpture. Apart from the deadline, Reid had absolute freedom to decide when and how long to work. CCNV paid Reid $15,000, dependent on completing the job (not a salary). Reid controlled hiring and paying assistants. Creating sculptures was not CCNVs regular business. CCNV did not pay payroll or social security taxes to Reid. 8) Note: Notes 6 and 8 (pp. 275-276 were covered in class, but not assigned. Cover them?

Joint Works
Rule: 1) Both authors must intend, @ time each co-authors work was created, that A) their contributions be merged into inseparable or interdependent parts of a unitary whole, and 2) B) that there was sufficient evidence to permit a reasonable trier to find that Childress had the requisite intent 3) Each putative (supposed) joint author must submit an independently copyrightable contribution

Chapter 4. Ownership and Transfers

20/55

Childress v. Taylor (2 Cir 1991) (p. 280) Joint authorship analysis/test 1) Facts: Def Taylor = actress; wanted to write a play about Moms Mabley. Ptf Childress = actual playwright. Taylor gave all of her research to Childress. Childress wrote the play. But in additional to providing the research material, Taylor also discussed with Childress the inclusion of several scenes and characters. Childress wrote the actual play structure and dialog. Childress finished the play w/in an agreed time frame, and she filed for and received a in her name. A letter from Taylors agent to Childress agent said the finished play shall be equally owned and be the property of both Taylor and Childress. The relationship soured, and Taylor got a different playwright to write another play, using Childresss scrtipt. 2) Procedure. Childress sued Taylor (et al) for violations of Act, the Lanham Act, and NYs anti-dilution statute. Dist Ct held that Taylor was not a jt author, and granted sum judg for Childress. 3) RULE (Dist Ct): A work qualifies as a jt work under 101 ONLY WHEN both auths intended, @ time wk was created, that A) their contributions be merged into inseparable or interdependent parts of a unitary whole, and B) that there was sufficient evidence to permit a reasonable trier to find that Childress had the requisite intent 4) Issue: Is Taylor a joint author of the work? (e.g. Childress = dominant author, and Taylor = subordinate author?) 5) Holding: No. Childress is the sole author. 6) Rationale: (Use this language!) Each jt author must make independently copyrightable contributions to the joint work. Here, No evidence of mutual intent. Taylor & Childress had not signed a formal contract showing intent. Without a K, the copyright stays w/ the persons who created able material. Here, only Childress did that. Taylor only contributed ideas/research, but not a able contribution. If Taylor wanted to own the , she could have hired Childress as WMFH, or she could have contracted for Childress to assign the rights to her. 7) Note 7, p. 291: Intent Problems a) What if one auth is dominant, and the other is subsidiary. Say the subs auth makes non-de minimis able contributions to the work, but she cannot satisfy the mutual intent requirement because the dom auth absolutely, vehemently and totally rejects collaboration so that he can make the work his own? consider Thomson v. Larson

nd

b) Thomson v. Larson (p. 291) In determining jt work, consider objective criteria, such as Credit/Billing. c) In-class hypo: Say 2 different people contribute different whole SCENES to a play; each of which is . The dominant authors acceptance of the subsidiary authors scenes proves intent? The dominant argument would argue that the subsidiary author implicitly agreed to assign the rights to the dominant author Arguments FOR copyrightability requirement i) ii) i) ii) Greater administrative efficiency TEST = enables parties to predict whether their contributions Parties were actors in a theatre company; they were in a group writing plays The group broke down and only so many people contributed to the final product the others

8) Note 3, p. 290 Requiring contributions to be copyrightable: a)

b) Erickson v. Trinity Theatre (p. 290)

iii) Holding: The person who was at all the meetings owned the ; the rest of the people could not prove that their contributions were able 9) Note 2, p. 289 BUT EXCEPTION TO COPYRIGHTABLE CONTRIBUTION RULE a) Gaiman v. McFarlane (7th Cir. 2004) (p. 289)

Chapter 4. Ownership and Transfers

21/55

i)

Facts: McFarlane asked Gaiman to contribute a script for the comic book series Spawn for $100,000. Gaimans script (that McFarlane drew), included 3 new characters that greatly spurred sales & spin-off toys, etc. Procedure: Gaiman sued, arguing (among other things) jt authorship of the characters he contributed. McFarlane argued that, even in their final form, the characters were uncopyrightable

ii)

iii) Holding: Gaiman wins iv) Rationale/Dicta: Although Gaimans verbal description may have been of a stock character, once he was drawn and named and given speech, he became sufficiently distinctive to be copyrightable. Gaimans contribution may not have been copyrightable by itself, but the contribution had expressive content without which the character wouldnt have been a character at all, but just a drawing. 10) Note 4, p. 290 The Parties Intent a) Default Rule: is that the parties must have the intent AT THE TIME EACH CO-AUTHOR MAKES HIS CONTRIBUTION that theyll be jt authors i) Modified Rule (under Shapiro): Even if, at the time one element of a putative joint work was created there was no intent to contribute to a joint work, the final work can become a joint work if at any time the author agrees with another contributing author Facts: A composer created a song, not intending that it be accompanied by words. He assigned his rights to a publisher, which then commissioned lyrics.

b) BUT SEE Shapiro, Bernstein & Co v. Jerry Vogel Music Co (2d Cir. 1955) (p. 290)

ii)

iii) Holding: The final song was a joint work, even though the composer never intended for it to be. iv) BUT NOTE: This case has been overruled by the definition of joint work in 101. 11) Derivative Works of Joint Works If A + B make a joint work, and A makes a derive of the original, A owns only on the portions of the derive work original to him. a) NOTE: B does not generally have to get As consent to make a derive work, unless they agreed to that term beforehand. Joint authors (or the licensees of such an owner) cannot be liable to other co-owners for infringing the in the work each joint author is essentially a tenant in common: They own an equal, undivided interest in the whole work e.g. Jt auths can grant a non-exclusive license of the work to anyone w/o other authors consent. But the joint authors MUST AGREE on exclusive licenses. i) Facts: Davis had co-written a song w/ Chambliss (Bliges writer). Blige recorded the song w/o ever consulting Davis. Davis claims infringement. (Then afterwards?) Chambliss assigned rights to Miller to record for $1. Miller assigned the right to record to Blige. Chambliss was a jt author, so he had the right to transfer the right to record non-exclusively. Chambliss then properly splits the royalty on $1 with Davis (he cheated him).

12) Co-Authors Rights in the Copyright a)

b) Jt auths still are obliged to split profits w/ the other authors c)

d) Davis v. Blige (2d Cir. 2007)

ii)

iii) Holding: In favor of Davis cant do an after-the-fact conveyance of rights to deprive the Ptf of the rights. Also, bad faith. Davis had the right to sue Blige as though the assignment never took place. 13) Note 12, p. 293 Proof of Intent a) Intent can be proved by direct or circumstantial evidence b) Many courts look at by lines, co-authorship credits, etc

Chapter 4. Ownership and Transfers

22/55

c)

Janky v. Lake County Convention & Visitors Bureau (7th Cir. 2009) i) Facts: Janky composed a theme song for use by the Bureau. Farag, another band member, suggested changes, and they released a new version of the song w/ 10% of the lyrics changed. Janky got a new on the new song, listing Farag as co-author of a joint work. Also filed w/ ASCAP stating that Farag held a 10% ownership share. Later, Janky filed another application omitting Farag and listing Janky as the sole author. Holding: Janky cannot do thatshe had already documented co-authorship.

ii)

Collective Works and the Tasini Case


201(c) Collective work provisions 1) A collective work as a whole is copyrightable as a whole (eg in the selection and arrangement of the contributions). Individual contributions are copyrightable in and of themselves. Copyright in each separate contribution to a collective work is distinct from copyright in the collective work as a whole 2) Copyrights for separate contributions in a collective work belong to the individual authors of those contributions IF theyre works made for hire 3) Example: In the New York Times, each article and photograph has a separate copyright, and there is an additional copyright in the collective work as a whole (i.e. in the selection and arrangement of the contributions). The Times owns the copyright in the collective work as the publisher and compiler. The Times also owns the copyrights in the original contributions made by its employees (because the employees are working within the scope of employment). But freelance authors (independent contractors) own the copyrights in their own contributions. Default Rule for Copyright Ownership in collective works: 201(c) In the absence of an express transfer of copyright or of any rights under it, the copyright owner in the collective work is presumed to have acquired only the privilege of reproducing and distributing the contribution as part of a) that particular work, b) any revision of that collective work, and c) any later collective work in the same series Tasini v. New York Times, Co (SDNY 1997) 1) Facts: Tasini & other freelance authors contributed works to New York Times & 2 other periodicals. Under agreements w/ the periodicals publishers, but without the freelancers consent, LexisNexis and UMI placed copies of the freelancers articlesalong with all other articles from the periodicals in which the freelancers work appearedin 3 electronic databases. Issue: Is this use authorized by the default privilege in 201(c)? Procedure: Dist Ct ruled in favor of the publishers, holding that the databases preserved the selection of articles in the collective works, even if they did not preserve their arrangement. On appeal, 2d Circuit reversed, saying that the databases did not preserve the selection and arrangement of the collective works. Supreme Ct granted cert. Holding: Affirm 2d Circuit: Authors win -- 201(c) does not allow the use. The databases did not constitute a revision of the collective works in which the works were first published

2) 3)

4)

5) Rule: 201(c) Without an express transfer of from contributors to the publisher, the publisher can only copy/distribute the contribution as part of a) that particular work, b) any revision of that collective work, and c) any later collective work in the same series 6) Rationale: The majority read 201(c) narrowly and viewed the online databases as a new anthology or compendium, rather than as a revision of the individual periodicals. The databases did not preserve the visual/structural integrity (selection/arrangement), nor did they preserve the selection Dissent: A disk containing all the files from a given issue of the New York Times constituted an electronic revision. There is no reason to treat the same collection of files differently when it was combined w/ the contents of other editions. Policy: Authors contributing to a collective work expect their work only in the publication itself, and not to be given to others for the profit of the collective work publisher. The contributing authors want to have the chance to profit from their work.

7)

8)

Chapter 4. Ownership and Transfers

23/55

9)

Note: Publishers started insisting on all rights agreements designed to allow them to reuse freelancers contributions. Those are now standard. q) Greenberg v. National Graphic Society (11th Cir. 2001) (decided 3 months before Tasini) r) Holding: A CD-ROM containing exact, page-by-page digital reproductions of all the contents of all issues of the magazine, along with a computer program enabling users to navigate the contents and an animated introductory sequence of morphing cover images, constituted a new collective work, rather than a revision within the meaning of 201(c). Faulkner v. National Geographic Enterprises (2d Cir. 2005) (decided after Tasini) Holding: The court decided that Greenberg was wrongly decided, because the Complete National Geographic preserved the selection, coordination, and arrangement of the underlying works and presented the contents to its users in the same context as they were presented to the users in the original versions of the magazine.

10) See also:

s) t)

Transfers of Rights
owners have unlimited ability to alienate right (transfer, assign, etc)

Provisions for Transfer of Ownership


1) 201(d) Allows ownership of , or any part of it, to be transferred by any means of conveyance or by operation of law 2) Provides that is to be treated as personal property upon death of the owner 3) Provides that is divisible: any of the exclusive rights that make up a , including those enumerated in 106 and any subdivision of them may be transferred and owned separately

Distinction between Ownership of Copyright and Material Object


1) 202 Distinguishes between ownership of Copyright and material Object 2) Copyright ownership and ownership of a material object in which the copyrighted work is embodied are entirely separate. e.g. owning a copy of a book does not give you copyright ownership in the book.

Recordation
1) 204, 205 Execution and Recordation of Transfers 2) 204 a transfer of copyright ownership (other than one brought about by operation of law) is valid ONLY IF a) There is an instrument of conveyance, or alternatively a note or memorandum of the transfer, b) in writing, and signed by the owner or an authorized agent 9) 205(a) Any document pertaining to a copyright may be recorded under (a) if it bears that actual signature of the person who executed it, or if it is appropriately certified as a true copy. 10) 205(c) The recorded document will give constructive notice of its contents only if: a) the document or attached material specifically identifies the work to which it pertains so that a reasonable search under the title or registration number would reveal it, AND

b) registration has been made for the work 11) 205(d) Priority Between Conflicting Transfers a) If two transfers conflict, the one executed first prevails if it is recorded in the manner required to give constructive notice under subsection (c) Assignee gets one month to record

Chapter 4. Ownership and Transfers

24/55

Effects Associates, Inc. v. Cohen (9 Cir 1990) 1) Facts: Def Cohen wrote a horror B movie. Def asked Effects Associates to create footage to enhance certain sequences in the film. EA agreed to prepare 7 shots. Cohen agreed orally, but no mention of who would own the in the footage. Cohen was unhappy w/ the effects, and he paid EA of the agreed amount. But he used the footage in his movie.

th

2) Procedure: EA sued for infringement. Dist ct dismissed on the ground that the suit was a contract suit, and not a copyright suit. 9th Circuit reversed and remanded, concluding that the ptf was the master of his claim and could opt to sue for infringement instead of suing on the contract. On remand, the issue was: Did EA transfer the right to use the footage to Cohen? The dist ct granted sum judg to Cohen, holding that EA had granted Cohen an implied license to use the shots. 9th Cir reviewed the dist cts ruling 3) 5) Issue: Did Cohen have the right to use the footage? Rationale: The court analyzed 2 issues: 1) Copyright Issue: EA is the copyright owner of the special effects footage used in the movie, and def Cohen copied, distributed, and publicly displayed the footage w/o written authorization. The agreement between Cohen and EA was not written. Under 204, the purported assignment is not valid. BUT 2) Non-Exclusive Licenses: Cohen had a non-exclusive license to use the footage. A non-exclusive license may be given orally, or even implied by conduct. Citing Oddo, the court reasoned that EA created a work at Cohens request, and handed it over, intending that Cohen copy and distribute it. This constitutes an implied grant of a non-exclusive license. But Cohen may sue for contract claims in state court. 4) Holding: Yes. Cohen wins summary judgment on claim.

Chapter 5. Duration and Terminations


Duration of Copyrights
302 Duration of Copyright in Works Created After Effective Date (of 1976 Act, i.e. Jan 1, 1978) 1) 2) 3) 4) 1) Basic Copyright Term: A work created on or after Jan 1, 1978 is protected from the date of its creation to the end of the authors life + 50 years (pre Sonny Bono Copyright Termination Extension Act (CTEA) of 1998) Alternate term For Anonymous Works, Pseudonymous Works, and Works Made for Hire: 75 years from publication or 100 years after creation (e.g. for a posthumous work??) General Rule: The Life + 50 term would apply equally to unpublished works, works published during the authors lifetime, and to works published posthumously. Joint Works: Term = life of last survivor + 50 years Every original work of authorship fixed in tangible form that is in existence would be given statutory copyright protection as long as the work is not in the public domain in this country

303 Preexisting Works Under Common Law Protection

See policy arguments on p. 323 Add in joint works & stuff pp. 324 - 326

Duration Basics Under the CTEA (Copyright Termination Extension Act)


Eldred v. Ashcroft (U.S. 2003) 1) Facts: Congress has extended the duration of copyrights several times (in 1831, 1909, 1976, and 1998 CTEA). Eldred (P) builds off of copyrighted works after they go into the public domain and sued Ashcroft (D), claiming that the CTEA violated the constitution. Eldred (P) alleged that Congress (D) did not have the power to expand

Chapter 5. Duration and Terminations

25/55

the term for published works with existing copyrights, and that the CTEA is a content-neutral regulation of speech that fails heightened judicial scrutiny. 2) Eldred (P) challenged Congress (D) authority to expand the term for published works with existing copyrights. 3) Procedure: The dist ct and the court of appeals found the CTEA to be valid. 4) Issue: 1) Does CTEAs extension of existing s exceed Congresss power under the clause? 2) Does the st CTEAs extension of existing and future s violate the 1 Amendment? 5) Holding: No and No. Congress(D) has the authority to enact legislation that extends the copyright terms for both existing and future copyrights (Affirm dist & appellate court. Congress wins. ) 6) Rationale: 1) The provisions of CTEA do not violate the Constitutions limited Time prescription because the text of the Constitution, as well as history and precedent, allow for such an extension. Furthermore, the CTEA st is a rational exercise of legislative authority. 2) CTEA does not violate the 1 Amendment because strict scrutiny should not be applied. The Copyright Clause already incorporates its own speech-protective safeguards. e.g. the Copyright Clause makes only expression eligible for protection; thus, every idea in a copyrighted work is available for public exploitation. Moreover, the fair use defense allows the public to use the expression contained in a copyright work in some circumstances. 7) Dissent (Stevens): The majority mistakenly believes that Congress (D) actions under the Copyright Clause are judicially unreviewable. The majority improperly relied on the history and the 1831 amendment because Congress based its authority to pass the amendment on grounds later declared improper by the Court. Moreover, precedent contradicts the cases relied on by the majority for support that extended terms were upheld historically. Things should go into the public domain. A rule prohibiting the extension would preclude a perpetual copyright. 8) Dissent (Breyer): Breyer dissents because CTEAs benefits to the private sector outweigh the public benefits (and indeed the CTEA harms the public). The extension will not act as an economic incentive encouraging authors to create new works. Furthermore, CTEA does not create a uniform American-European term as the American Act produces an extended term of 95 years while comparable European rights in for hire works last for periods that vary from 50 years to 70 years to life plus 70 years. In addition, CTEA does not provide an incentive to publishers to republish and to redistribute older copyright works because the Clause assumes that it is the disappearance of the monopoly grant that will promote the dissemination of works already in existence. CTEA also refers to the financial assistance it will bring to the entertainment industry. But this is commerce, not copyright

The Law of Renewal


The law of renewal under the 1909 Act is still relevant for works first published or registered before 1978

Renewal Basics
The idea of renewal is to give authors a second bite at the apple and perhaps renegotiate bad deals to capitalize on a successful work.

Transfer of the Renewal Term


a) Fisher undermined the basic policy of the renewal grant, which was to protect the unequal bargaining position of many authors i) Holding: The assignment by an author of the renewal term, before the right thereto had vested, was binding on the author. i.e. if B can get A to assign the rights of the renewal term before its vested, A has to live with that. b) Fred Fisher Music Co. v. M. Witmark & Sons (U.S. 1943) (p. 348 note case)

Chapter 5. Duration and Terminations

26/55

ii)

Rationale: The majority rejected the argument that authors must be protected against bad deals. If an author cannot make an effective assignment of his renewal, it may be worthless to him when he is most in need. Nobody would pay an author for something he cannot sell. The majority advanced the policy of freedom of contract. Dissent: This goes counter to the purpose of the renewal term the author should be able to renegotiate a bad deal. Note: After Fisher, it became industry practice to require an assignment of the authors renewal rights in the initial contract, which took away from the rights of authors.

iii) iv) a)

The author had to survive until the renewal term vested (from Miller Music Corp v. Charles N. Daniels, Inc. (U.S. 1960) note case p. 348) i) ii) An authors assignment of the renewal term was a CONTINGENT interest only. i.e. If an assigning author died before the renewal vested, the right to the second term vested not in the assignee of the first term, but rather in the authors statutory successors under 24 of the 1909 Act.

Renewal and Derivative Works


Fill in Stewart v. Abend

Automatic Renewal
Fill in p. 353

Golan v. Holder - US usage of foreign-copyrighted material??


Golan v. Holder (U.S. 2011) Facts: 12) In the early/mid 20 century, many foreign works prematurely entered the public domain in the US because of failures to comply with US formalities for protection, such as publication in the US. For many years, other countries did not have the same level of formalities that the US had (e.g. notice). Many foreign authors inadvertently lost their s to the public domain. 13) The US joined the Berne Convention in 1989the BC provided a mimimum level of copyright protection for signatory countries. It was thought that as part of joining Berne, the US would restore protection to the works lost to the public domain. But US didnt do that shit. 14) But then, US joined NAFTA/TRIPS in 1994, which created an enforcement mechanism (under the World Trade Organization). 15) Restoration Act of 1996 would restore protection to authors who lost protection for 1 of 3 reasons: 1) Author had not complied w/ US formalities; 2) sound recordings recorded before the US gave protection to sound recordings in 1972, 3) the work originated in a country with which the US did not have provisions (e.g. former Soviet Union). 16) Had to meet 3 reqs for restoration: 1) Work must have protection in the source country; 2) Author must have been a natl or domiciliary of an eligible country at the time the work was created. If work was published, it must have been published in an eligible country; 3) The work must have fallen into the public domain for the reasons above (e.g. foreign work, lack of formalities) 17) A group of artists, including musician Lawrence Golan, who made use of the works while they were in the public domain filed a lawsuit in Colorado's federal court to challenge the restoration of copyright, arguing that doing so violated their First Amendment rights. Rule:
th

Chapter 5. Duration and Terminations

27/55

18) 104A in Copyright Act (a.k.a. 514 in the Copyright Restoration Act) protects reliance parties -- Parties who own copies of works, and parties who prepare derivative works of the works that fell into the public domain. RPs only responsible for s if the original owners contact them. 19) Lost- holders (e.g. Soviet author) could serve blanket notice or actual notice the RP could sell works for 12 mos; RPs w/ Deriv Wks could make them beyond 12 mos if pay reasonable compensation 20) Note: Balances the rights of the lost- holders with the Reliance Parties 21) Procedure: Suit brought against Restoration Act both under and First Amendment 22) Holding: The CRA did alter the contours of protection (going back to Eldred v. Ashcroft). Because in our country, usually once in pub-dom, always in pub-dom. That triggers First Amendment issues 23) Triggers Intermediate Scrutiny -- Govt now has to show furthering an important govt interest, and narrowly tailored to effectuate that end 24) Colorado Dist Ct Restoration Act is too broad; does not protect RPs enough; it fails Int Scrut. The U.S. District Court for the District of Colorado held that Section 514 of the URAA does not violate the Copyright Clause or the First Amendment. The district court also rejected Golan's First Amendment challenge, seeing "no need to expand upon the settled rule that private censorship via copyright enforcement does not implicate First Amendment concerns." 25) 10 Circuit Thinks it passes Int Scrut important govt interest, and narrow. The United States Court of Appeals for the 10th Circuit affirmed in part and reversed in part. The court agreed that Section 514 of the URAA does not exceed Congress' authority under the Copyright Clause, but it vacated the district court's First Amendment ruling and remanded for further proceedings 26) Supreme Ct Affirmed the lower court (6-2) 27) Issues: a) (1) Does the Progress Clause of the U.S. Constitution prohibit Congress from taking works out of the public domain?
th

b) (2) Does Section 514 of the Uruguay Round Agreements Act violate the First Amendment of the United States Constitution? 28) 1) No and 2) no. In a 6-2 decision, Justice Ruth Bader Ginsburg wrote the majority opinion affirming the lower court. a) 1) The Court held that congress did have the power to enact Section 514 and the Uruguay Round Agreement. An exhaustive recount of the history of copyright law showed that the public domain is not untouchable, and congress may grant protection to previously unprotected works. The Court soundly rejected Golan's argument that taking works out of the public domain violated the constitutional "limited times" requirement, following the reasoning from Eldred v. Ashcroft, 537 U.S. 186 (2003).

b) 2) The Court also held that there were sufficient First Amendment protections built into the current copyright law in the form of the idea/expression dichotomy and the fair use doctrine. 29) Dissent: Justice Steven G. Breyer wrote a dissent, stating that the statutes in question violated the Constitution because they did not promote the production of new works. Justice Beyer also expressed concerns about the difficulty of finding authors of orphaned works in order to gain a copyright license.

Terminations of Transfers
Authors may transfer rights to others during the terms of their s. Sometimes, if the work achieves large commercial success, the author feels undercompensated for the rights transferred. Note: The 1909 Act did not do as well to protect authors (see e.g. Fisher). The 1976 Act purposefully did. 203, 304(c), 304(d) Give an author and her family a right to terminate if they wish, but require compliance w/ procedures established by statute and regulation to effect the termination. Termination is NOT automatic. If the author fails to take the steps to terminate the grant of transfer, then the transfer continues.

Chapter 5. Duration and Terminations

28/55

203 Terminations Post-1977 Transfers (see pp. 362 366) 1) 2) 3) Purpose of 203: To provide an author and her family a second chance to reap the benefits of the authors creations by giving the right to terminate transfers made during the copyright term. 35 years from time of grant, a window opens. From years 35-40 (5 years), the author can terminate the grant. 203 applies only to TRANSFERS made after 1977. It does not matter when the COPYRIGHT came into existence. It only matters when the TRANSFER was made. If the work was created before Jan 1, 1978, and the transfer was made before that date, then 304(c) applies. Notice Rules: Must be served at least 2 years and no more than 10 years before the effective date of the termination 203(a)(5) Grants can be terminated in spite of contract; i.e. you cant contract away the rights to terminate. Note: The very first grants subject to 203 were made in 1978, so 2013 is the first year that the 5 year window opens. Well see what happens. Fill in

4) 5) 6)

304(c) Terminations: Pre-1978 Transfers 1) 304(d) Terminations Fill in

The Mechanics of Termination


5 Step Process
1. Determine which section of the statute ( 203, 304(c), 304(d)) applies to the grant to be terminated a) 203 applies to all grants executed by the author on or after Jan 1, 1978. It is the date of the TRANSFER to be terminated, not the date of the COPYRIGHT, that determines 203.

b) 304(c) and 304(d) apply only to transfers of the renewal period, executed before Jan 1, 1978, in works copyrighted before Jan 1, 1978. c) 304(c) and 304(d) apply to grants made by the author, authors widow, widower or children, and next of kin

d) 304(d) is limited to works whose copyright was first secured between Jan 1, 1923 and Oct 26, 1939 (i.e. it only applies to works that are in their renewal term as of Oct 27, 1998, the effective date of the CTEA). e) 2. 304(d) applies only if the owner of the 304(c) termination right has not previously exercised that right 304(c)(3) the 5-year termination window opens at 1) the end of the 56 year period (on the anniversary of the copyright date, not the following Jan 1), or 2) Jan 1, 1978, whichever is later (see Emanuels p. 192)
th

Calculate when the termination window opens and closes. a)

b) 304(d) the 5 year termination window opens on the 75 anniversary of the date the copyright was originally secured. c) 203(a)(3) Generally: The 5 year termination window generally opens 35 years after the date of the transfer to be terminated. If the transfer includes publication rights, the 5 year window is postponed to the earlier of 35 years after publication or 40 years after the date of the transfer Under all 3 statutes: Notice must be served at least 2 years before the effective date i) The latest date notice may be served is 2 years before the date that the 5 year termination window opens

3.

Select an effective date within the termination window a)

Chapter 5. Duration and Terminations

29/55

ii)

If that fails, the effective date must be at least 2 years after you plan to serve notice. There must be at least 2 years left before the 5 year window closes (i.e. if there are fewer than 2 years remaining in the termination window, it is too late to serve notice).

4.

Service notice NO MORE THAN 10 YEARS and NO LESS THAN 2 YEARS before the effective date a) Under all 3 statutes: Termination must comply with Copyright Office regulations. b) Notice must be signed by the number and proportion of persons entitled to terminate, or their duly authorized agents. c) Grants Made Before 1978 304(c)(1) and 304(c)(2) The persons who are entitled to serve notice for terminations governed by 304(c) and 304(d) i) 304(c)(1) Grant executed by someone other than the author: May be terminated by the surviving person or persons who executed the grant. Grant executed by one or more authors of the work: May be terminated by the author who executed it (or by authorized people who own more than of the authors termination interest see also subsection 2) 304(c)(2) Where an author is dead: A) The widow or widower has 100% of the termination interest, unless there are any surviving children or grandchildren. In that case, the widow or widower owns of the interest. B) The authors surviving children, and the surviving children of any dead child of the author, own 100% of the interest, unless there is a widow or widower. In that case, the ownership of of the authors interest is divided among them. C) etc (see supp. p. 135) iii) Grant made by Joint Authors: The authors are tenants in common and each may terminate separately to the extent of his/her interest. d) Grants Made After 1977 203(a)(1) and 203(a)(2) i) For Living Authors (a)(1): A) One Author: May be terminated by the author. B) Two or more Authors: May be terminated by a majority of the authors who executed it. If any of such authors is dead, the termination interest may be exercised by the person or persons (under subsection 2) who own and are entitled to exercise a total of more than of the authors interest ii) e) For Deceased Authors (a)(2): same as the the provisions for 304(c) and 304(d).

ii)

5.

Record a copy of the notice in the Copyright Office before the effective date a) On the effective date, the rights covered by the terminated grant revert to the people who were entitled to terminate on the date the notice of termination was served. iii) 304(c) Each authors interest is owned separately from the interests of any co-authors iv) 203 It takes a majority vote of the joint authors to terminate or to make a further grant

b) Joint Authors

Siegel v. Warner Brothers Entertainment, Inc (C.D. Cal 2008) (p. 375) 1) Facts: Siegel & Shuster created Superman character. They assigned exclusive rights to D.C. Comics. 1 comic published w/ Superman was called Action Comics (in 1938). Superman became an instant success. Siegel and Shuster sued twice to annul and rescind their previous agreements assigning their ownership rights in Superman. The courts held that Siegel and Shuster had signed away their rightsboth copyright and renewal rights. But then the 1976 Act gave artists and their heirs the ability to terminate any prior grants of the rights to their creations that were executed
st

Chapter 5. Duration and Terminations

30/55

before Jan 1, 1978, regardless of the terms contained in such assignments. They issued 7 termination notices. Even though subsequent Superman works feature facets of Superman not originally conceived by Siegel and Shuster (e.g. xray vision, ability to fly) and additional characters, the heirs included in their termination notices that the notices applied to each and every work that includes or embodies Superman. The effective date of termination was 4/16/1999. 2) Procedure: Siegels wife and daughter seek a declaration from the court that they have terminated the 1938 grant by Siegel and his creative partner, Shuster, of the copyright in their creation of Superman. They aim to recapture Siegels half of the Superman copyright. Issue: Do Siegels wife and daughter have rights, as heirs, to terminate the assignments and recapture Siegels half of the copyright share? Holdings: Ptfs are able to regain Siegels copyright interest, with limitations. 1) Promotional Announcements: Going back 61 years from the termination effective date (56 years + 5 year window?), = 4/16/1938... At least one of the comics containing promotional announcements with Superman was published before the 4/16/1938 cutoff, and defs could use that however they chose. That picture was somewhat nondescript, anyway (see p. 381). 2) Work Made for Hire Aspect of Portions of Action Comics, Vol. 1: The Superman material in Action Comics Vol 1 is not a WMFH, and it may be terminated. Defs do not own Siegel and Shusters pre-existing Superman material, because S&S had already created it before they were hired by DC Comics. (The reasoning had been applied in a prior case, so it was rejected here by collateral estoppel.) Rule Under the 1976 Act, an author may not terminate a prior grant in a work made for hire, because the right to grant is not theirs to begin with (it belongs to the employer see 304(c)). There was some material that was submitted re-submitted in a reformatted version. 4) The Ptfs can only claim the domestic portions of the , and they are not entitled to any foreign profits generated by DCs use of Superman 5) Note: This was an observation, but not part of the decision. The court could not decide because the record was too narrow: 1) Per 304(c)(6)(E), the Ptfs may not be entitled to any profits related to DCs trademarks in Superman, even though they used copyrighted material. Per 304(c)(6)(A) Derivative works created during the grant may be continue to be exploited after termination

3) 4) 5)

6)

7) 3) The Ptfs did not fail to identify all grants to which the termination notice applies 8) 9)

Chapter 6. Publication and Formalities


Publication
Fill in (pp. 393-396)

Notice
Fill in (pp. 409-410)

Deposit and Registration


Fill in (pp. 420-422)

Chapter 7. Exclusive Rights and Their Limitations


106 Exclusive Rights
1) Reproduction: To reproduce the copyrighted work in copies or phonorecords 2) Adaptation: To prepare derivative works based on the copyrighted work

Chapter 7. Exclusive Rights and Their Limitations

31/55

3) Public Distribution: to distribute copies or phonorecords of the copyrighted work to the public by sale or other transfer of ownership, rental, lease, or lending 4) Public Performance: to perform the copyrighted work publicly a) Note: This applies to literary; musical, dramatic, and choreographic; pantomimes; and motion pictures and other audiovisual works it does NOT apply to pictorial, graphic or sculptural works (probably because it wouldnt make sense to) Note: This applies to literary; musical, dramatic, and choreographic; pantomimes; pictorial, graphic, or sculptural; and motion picture or other audiovisual works (e.g. to display individual images of a movie)

5) Public Display: to display the copyrighted work publicly a)

6) Public Performance by Digital Audio Transmission: This applies only to sound recordings 7) Note: The rights in 106 are limited by 107-122. For example

Statutory (or Compulsory) Licenses


A 3 party can use a copyrighted work w/o the copyright owners permission, as long as he complies w/ the statutory procedure and pays the established royalties. 1) The Cable TV License ( 111) Statutory license for secondary transmissions by cable tv systems 2) The Ephemeral Recordings License ( 112(e)): Statutory license for ephemeral recordings used to facilitate the digital transmissions permitted under 114 3) The Digital Performance Right in Sound Recordings License ( 114): Statutory license for some operators of non-interactive digital transmission services, AS LONG AS those copies or phonorecords are not commercially distributed by or through public broadcasting entities to the general public b) 114(b) -4) The Mechanical License ( 115): Statutory license for the reproduction and distribution of phonorecords of nondramatic musical works, (and now includes provisions that apply to authorized digital phonorecord delivery of such musical works a) Note: This overrules the White-Smith ruling (see p. 6 for White-Smith decision). Anyone may cover an original work (as long as they pay the royalty and follow the rules)
rd

b) MUST be for private use AT HOME, and ONLY APPLIES TO NON-DRAMATIC MUSICAL WORKS 5) The Public Broadcasting License ( 118): Statutory license for the use of certain copyrighted works by noncommercial broadcasting entititses (e.g. public radio) 6) The General Satellite Retransmission License ( 119): Statutory license for satellite retransmissions to the public for private viewing 7) The Local-to-Local Satellite Retransmission License ( 122): Statutory license for satellite retransmissions of local television stations broadcasts into their local markets. 8) 114 and 115 Compared: a) 114 limits reproduction, adaptation and performance rights in SOUND RECORDINGS. i) ii) 114(a) limits the exclusive rights in sound recordings to reproduction (106(1), adaptation 106(2), distribution (106(3), and public performance via digital transmission (106(6). 114(b) Limits the exclusive rights in sound recording to ONLY THAT SOUND RECORDING. i.e. the reproduction (106(1) and adaptation (106(2)) rights do not extend to other sound recordings. Others may make another sound recording that sounds exactly like the protected one, and the (C) owner cannot protect that independently created work.

iii) 114(b) The reproduction (106(1), adaptation 106(2), distribution (106(3) rights do not apply to sound recordings included in educational TV and radio programs distributed or transmitted by or through public broadcasting entities (e.g. PBS) AS LONG AS they are not commercially distributed.

Chapter 7. Exclusive Rights and Their Limitations

32/55

b) By contrast, 115 limits the reproduction, adaptation and distribution rights in certain MUSICAL WORKS (non-dramatic) through the creation of a compulsory license

The Reproduction Right


Fill in (pp. 466 77) 11) The owner of the reproduction right may make her own copies or phonorecords of the work and prevent others from doing so w/o her permission. a) Only applies to material copies or phonorecords merely reading a work aloud does not infringe (or in the case of choreography, dancing does not infringe) though this may constitute a violation of the right of public performance i) ii) iii) iv) v) 113 Useful Articles (Pictorial, Graphic, and Sculptural Works) limits protection of ed works that are incorporated in useful articles see the case about the Lamp 120 Architectural Works permits making/distributing/displaying works (paintings, photos, other P/G/S works) as long as the bldg where the work is located in or ordinarily visible from a public place 108 Safe harbor for libraries/archives 112 allows broadcaster of programs lawfully containing performances or displays of ed works to reproduce their programs in the course of business 118(d) allows public broadcasters to reproduce programs they make containing published nondramatic musical works and published P/G/S works and distribute them for rebroadcast by other public stations 115 Mechanical license allows users to make and publicly distribute sound recordings of certain copyrighted nondramatic musical compositions if 1. 2. 3. 4. The owner has already made or authorized a recording of the work, which has been distributed to the US public The Prospective licensees plans are for private, not commercial use The prospective licensee serves notice of intention to use the compulsory license within a specified time The prospective licensee pays royalties to the owner (per 115)

b) Exceptions to Reproduction Right:

vi)

Reproduction in Phonorecords Copyright holder owns the right to control reproductions in phonorecords material objects, such as tapes, CDs that embody sound recordings The right to reproduce in phonorecords is limited (in common with the general right to reproduce other works) by 107, 108, 112, 112, and 121. Right to reproduce phonorecords is also limited by specialized limitations: 114 and 115. (see pp. 467 468) Cartoon Network v. CSC Holdings, Inc. (2 Cir 2008) (p. 470) 1. Facts: Cablevision's DVRs hold recordings at CSC Holdings--the home user does not have any recorded content at their home. Ptf supplies TV content to Def, for distribution. Def uses DVRs that record TV shows with 2 buffers: Primary buffer -- captures a second at a time, waits for viewer to decide to record it; once that happens, then the secondary buffer streams the show. If the viewer decides to record the show, then the show is recorded from that point in time, and stored online
nd

Chapter 7. Exclusive Rights and Their Limitations

33/55

2. 3.

Procedure: Dist ct ruled against Def. Held that the buffering was the embodiment of a copy. Issue: By buffering the data that make up a work, does Cablevision "reproduce" the work "in copies?" (if so, that would constitute copyright infringement) Holding: The def did not violate the ptfs right of reproduction Rationale: App Ct reversed -- 2 criteria: 1) Embodiment, and 2) Duration -- here, def isn't making a copy because it isn't developing an object that fixes the work for more than a transitory duration . Dist Ct had relied on MAI Systems case -- computer repair co would go into offices to repair computers; would load rd software into computers. The Ptf wanted to reserve all repairs for itself, and keep out 3 parties. The computer program was loaded into RAM. MAI held that storing the program in RAM was a copy for the purpose of infringement analysis. But the MAI Systems holding only looked at one criterion embodiment. It didnt look at duration. Another issue: Is an electronic reproduction a reproduction in a copy. Yes -- electronic reproduction CAN BE a copy (it is embodied) Another issue: Def's participation in the home taping by the viewer. Def is providing the mechanism for copying to take place. Is def committing (C) infringement? No -- Cablevision is not actively participating in the copying. It would be like charging Kinko's with (C) infringement because a customer copies a copyrighted book using a copying machine. Rule: 1) The work must be A) fixed (embodied in a medium), and B) sufficiently permanent for more than a transitory duration

4. 5.

6. 7.

8.

The Adaptation Right


Fill in 483-493 ( 106(2)) grants the right to prepare derivative works derivative works based on the copyrighted work. A derivative is a work based on one or more preexisting works, such as translation, musical arrangement, dramatization, fictionalization, motion picture version, sound recording, art reproduction, abridgement, condensation, or any other form in which a work may be recast, transformed, or adapted. Lee v. A.R.T. Company (7 Cir 1997) 1) Facts: Lee (P) creates works of art that she sells through her firm, Annie Lee & Friends. Deck the Walls, a chain of outlets, buys her works, which are registered with the Register of Copyrights. Deck the Walls sold some of P's work to A.R.T. Company (D), which mounted the works on ceramic tiles and resold them. P sued for monetary and injunctive relief, claiming these works were derivative works and could not be prepared w/o the permission of the Copyright proprietor. Procedure: The dist ct entered summary judgment for D. The dist ct held that the ceramic tile did not constitute an "original work of authorship" because it was no different than placing the picture in a frame or case. P appealed. Issue: For a work of authorship to constitute a derivative work, must it possess a sufficient level of creativity? Holding: Yes, for a work of authorship to constitute a derivative work, it must possess a sufficient level of creativity? Rule: A derivative work must either be in one of the forms named in 101, or be recast, transformed, or adapted. Rationale: This court believes that the mounting process does not create a derivative work because the change to the work was not sufficiently original. The mounting here did not constitute a transformation of P's work, since the work was not changed by the process. The court holds this, even though courts have held that framing an other methods of mounting do not infringe the author's right to make derivative works. The Ninth Circuit has previously held that D's method of bonding the art to the tile with epoxy resin creates a derivative work. a) Issue: Does a collectors guide to ptfs Beanie Baby toy line constitute a derivative work based on the designs of those stuffed animals?
th

2) 3) 4) 5) 6)

7) Note 7, p. 491 Ty, Inc. v. Publications, Intl.

b) Holding: No, it does not

Chapter 7. Exclusive Rights and Their Limitations

34/55

c)

Rule: A derivative work must either be in one of the forms named in 101, or be recast, transformed, or adapted. (citing Lee v. ART)

d) Analysis: The textual portions of a collectors guide to copyrighted works are not among the examples of derivative works listed in the act, and they dont recast, transform, or adapt the things to which they are guides. The guide is critical, evaluative, and purely informational. Ownership of a copyright does not confer a legal right to control public evaluation of the copyrighted work.

The Public Distribution Right


106(3) the right to distribute copies or phonorecords of the work to the public by sale or other transfer of ownership, or by rental, lease, or lending. Limitations to Public Distribution Right DEFENSE to public distribution: First Sale Doctrine ( 109) Once the owner transfers, or authorizes another to rd transfer, title to a copy/phonorecord of a ed work to a third party (e.g. to a retail store), the 3 party and his successors in interest are entitled to sell or otherwise dispose of it without the owners consent. note: ONLY APPLIES TO LAWFUL COPIES AND PHONORECORDS 602 Prohibits importation into the US of copies of a work acquired abroad w/o authority of the owner Domestic Distribution London-Sire Records, Inc. v. Doe 1 (D Mass 2008) About 40 John Doe copyright infringement complaints most naming multiple unidentified defendants were consolidated before the court. Record companies that contended students were sharing copyrighted music files on peer to peer sharing systems served Rule 45 subpoenas on a non-party university seeking information that would identify the students. Three students entered objections. Copyright infringement clearly was not entitled to First Amendment protection, according to the court. However, there were some creative aspects to sharing music files such as identifying with particular music or with those who associated with particular music that warranted minimal First Amendment protection. Here, it was possible that the record companies were being more intrusive than necessary to obtain identities of students. In one case before the court, the record companies sought a mirror image of a students hard drive for forensic examination. The court ordered modification of the two contested subpoenas to require the university to submit its responses under seal to the court for in camera inspection. The court also ordered the university to provide the court with a list of IP addresses from the record companies which the university could not easily attribute to particular students due to the universitys dynamic assignment of IP addresses by session on a computer rather than to particular computers. The modified subpoenas also required the university to submit its service agreement to the court for evaluation of privacy expectations of the students.

The Public Performance Right


106(4) the exclusive right to perform the work publicly. Note: This does NOT APPLY to 1) Pictorial/Graphic/Sculptural works; 2) Sound Recordings; 3) Architectural Works o 106(4) Radio/TV stations that broadcast ed sound recordings can do so with no obligation to the owner of the in the SOUND RECORDING, but they must get the rights from owners of the in the MUSICAL WORK that is recorded (from ASCAP/BMI). ASCAP/BMI offer blanket licenses SEE PUBLIC DISPLAY FOR LIMITATIONS ON PUBLIC PERFORMANCE AND DISPLAY
rd

Columbia Pictures Industries, Inc. v. Aveco, Inc. (3 Cir 1986)

Chapter 7. Exclusive Rights and Their Limitations

35/55

1.

Facts: Producers (P) of motion pictures brought a copyright infringement action against Aveco (D), claiming that D's business of renting video cassettes of motion pictures in conjunction with rooms in which they may be viewed violated their exclusive rights to publicly perform the cassettes. Procedure: The dist ct concluded that Aveco (D) infringed the Producers' (P) exclusive rights of to publicly perform and authorize performances of the copyrighted works and granted P's motion for partial summary judgment. Aveco (D) appealed. Issue: Does the renting of rooms to members of the general public in which they may view copyrighted video cassettes constitute the authorization of a public performance in violation of the copyright owner's exclusive right to publicly perform the work? Holding: Yes. The renting of rooms to members of the general public in which they may view copyrighted video cassettes constitutes the authorization of a public performance in violation of the copyright owner's exclusive right to publicly perform the work. (affirmed dist ct). Rule: 1) To "perform" a work is defined by 101 of the 1976 Act as "in the case of a motion picture or other audiovisual work, to show its images in any sequence or to make the sounds accompanying it audible"; 2) The Act defines the term "to perform a work publicly" as: "(1) to perform or display it at a place open to the public or at any place where a substantial number of people outside of a normal circle of a family and its social acquaintances are gathered; or (2) to transmit or otherwise communicate a performance or display of the work to a place specified by clause (1) or to the public, by means of any device or proces, whether the members of the public capable of receiving the performance or display receive it in the same place or in separate places at the same time or at different times." Rationale: Ps did not argue that D itself performed the cassettes, but that it unlawfully authorized the performances by enabling its customers to perform the videos in their viewing rooms. The performances constitute an infringement only if they are public. Aveco (D) authorized the public performances of the video cassette by renting room to members of the general public in order to view performances of the Producers' (P) copyrighted works. D was not protected by first-sale doctrine because FSD (see 109) applies only to distribution, not public performance Note 5, p. 526 Hotel Cases (public performance?) a) Taking ONE copy of a videotape to your hotel room = home viewing = private performance (Columbia Pictures Industries, Inc v. Professional Real Estate Investors, Inc)

2.

3.

4.

5.

6.

7.

b) BUT picking from a centralized list (e.g. OnCommand video) = public viewing (On Command Video Corp. v. Columbia Pictures Industries) 8. Note 2, p. 525 Is a Public Board Game a Public Performance? a) No well, we say no (Allen v. Academic Games League of America) b) I would argue that the board game is not a public performance because you cannot copyright every possible outcome of the sequence of the game play. 9. Note 7, 525 Public Performance and Transmissions a) Clause (2) of the 101 definition of publicly (supp p. 12) encompasses transmissions to a place listed in clause (1), and transmissions directly to the public, whether the members of the public capable of receiving the performance or display receive it at the same place or in separate places and at the same time or at different times.

b) Clause (2) is broad enough to encompass asynchronous transmissions, such as a video on demand service that transmits performances of audiovisual works to private homes. c) In Cartoon Network v. CSC Holdings (see above, p. 33), the 2d Cir rejected the ptfs argument that transmission of the stored programs to customers violated the public performance right. The court reasoned that each such transmission should be treated as a separate private performance because 1) the members of the public capable of receiving the performance were limited to the household of the customer who recorded it and 2) individuals receiving the transmissions could individually control the transmission,

Chapter 7. Exclusive Rights and Their Limitations

36/55

Limitations to the Public Performance/Display Right ( 110)


1. 2. Fill in (Note 10, p. 527 also, see notebook) See Notebook 2 p. 6

Performing Rights Societies


1. ASCAP/BMI/SESAC a. i. ii. iii. iv. American Society of Composers, Artists, and Publishers (ASCAP) Founded by and operated for composers and publishers US-Only Not-for-profit Founded in 1914 to administer performance right. In 1939, radio was starting to boom. ASCAP wanted to renegotiate its 5% royalty rate and get 10% royalty from radio stations. Radio stations refused, so for 7 mos in 1939, ASCAP pulled all d music from the radio. Contract Lengths 1. 1 Year Writer agmt w/ Coterminous 1 Year publisher agmt Shawn Murphys preferred a. b. vi. vii. viii. ix. Licenses In Effect clause: when one signs w/ ASCAP, the compositions licensed will remain in the ASCAP repertoire for the duration of the license in effect Money is split 50/50 btwn writer & publisher

v.

b. Broadcast Music Incorporated (BMI) BMI founded by broadcasting industry Not-for-profit Founded in 1939 by Broadcasters -- BMI negotiated low rates (than ASCAP) better deals for radio stations Contract Lengths 2. 3. 4. 2 year writer agmt and 5 year publisher agmt Coterminous for 10 years Upside: Writers can sign just as writer, theyll pay both the pub and writer share to the writers affiliation only stuck for 2 years, BMI licenses to broadcasters, and artists can take their own music after the 2 years Downside: Any licensee you deal with will normally have publishing representation. This means youll have to set up a publishing entity; any songs registered through that co have to stay in the repertory until the pub co dissolves at the end of the K c. c. x. xi. xii. xiii. xiv. 2. songs could be there as long as 3 years after the writer leaves; BMI not required to pay on compositions that they represent in the same way

5.

Society of European Stage Authors and Composers (SESAC) SESAC is a for-profit company Used to represent/administer the Grand-Right / Musical Theater (e.g. right to take a composition and dramatize it, such as Green Day) (i.e. not part of the same governance as ASCAP/BMI) But now SESAC (in the 1930s) flipped their business to the Small Performance Right Can charge higher royalties Contract Lengths 6. 3 year writer and 3 year publisher SESAC pays advances!

Collection of $ for public consumption for non-dramatic uses for clubs, concerts, muzak, radio, and any other public performance of a song

Chapter 7. Exclusive Rights and Their Limitations

37/55

1.

All these uses require some type of licensing by a PRO, typically blanket licenses are used

Small and Grand Performing Rights (p. CB p. 531)


1. 2. 3. ASCAP/BMI/SESAC all license small performing rights Copyright owners license grand rights separately Small right = rights to perform non-dramatic musical works, and to perform individual songs from dramatic musical works, separate and apart from the dramatic context in which they first appeared (e.g. to perform an individual song from an opera. Grand Right =

4.

Synchronization Rights
5. 6. 7. Synch rights = The rights to use existing compositions to supply the background/soundtrack to a movie/motion picture Not mentioned in 106 Synch rights MUST BE OBTAINED DIRECTLY FROM THE HOLDER

Performance Rights in Sound Recordings


Fill in 535-536 Fill in notes 536-537

The Public Display Right


106(5) Right to display a work publicly (DOES NOT APPLY TO SOUND RECORDINGS OR WORKS OF ARCHITECTURE) Perfect 10, Inc. v. Amazon.com, Inc. (p. 556) Perfect 10 was an adult entertainment magazine that provided a subscription-only website. A number of independent, third-party web site publishers placed images obtained from Perfect 10's subscription-only area on their own websites, violating Perfect 10's terms of service and copyright. Google crawls, indexes, and caches websites on their internal servers so they can be accessed quickly. The sites crawled included many of these third-party sites containing infringing images. As part of their image search service, Google also provides thumbnail copies of the images that are being searched for so the user may see them before accessing the website. Furthermore, when a user selects an image from a Google search, a new page is accessed that includes the original website as well as a frame that contains information about the image and the thumbnail version of the image.[2] Perfect 10 believed the linking constituted instances of secondary copyright infringement, and the caching and thumbnails constituted direct infringement. Acting on this, Perfect 10 sent Google infringement notifications for nearly 4 years, eventually filing suit against both Google and Amazon for similar activities. Perfect 10 requested injunctions against Google and Amazon from linking to websites displaying Perfect 10's images and, in the case of Google, displaying the thumbnail images. Issues: 1) thumbnails; 2) inline displays of unauthorized images Holding: Thumbnails = direct infringement of public display right (but Google ultimately wins on Fair use)

Moral Rights
Moral rights vary from country to country, but generally include:
1) Right of integrity: The right to insist that the work not be mutilated or distorted; 2) The right of attribution: The right to be acknowledged as the author of the work and to prevent others from naming anyone else as the creator; and 3) The right of disclosure: the right to decide when and in what form the work will be presented to the public

Chapter 7. Exclusive Rights and Their Limitations

38/55

Protection outside the Copyright Act (Intellectual Property)


Section 43(a) of the Lanham Act (trademark) Fill in pp. 573-575 State Art Preservation Laws

Protection under Copyright Law: The Visual Artists Rights Act of 1990 (VARA)
4) VARA Outline is in Book see CB p. 578 its highlighted 5) Works Protected: Visual Art only (e.g. photos, sculptures). Architectural works are NOT works of visual art under VARA. DOES NOT COVER WORKS MADE FOR HIRE Phillips v. Pembroke Real Estate, Inc. (1st Cir 2006) 1) Facts: PL/artist was a sculptor that Pembroke hired to do work for a park that they had redesigned. Phillips was commissioned by Pembroke to create 27 sculptures in a Boston park. Pembroke later decided to redesign the park, which would remove or relocate almost all of Phillips' work. 2) Procedure: The Dist Ct found that VARA applied to Phillips site-specific sculptures. But the court held that the art could be moved under VARAs public presentation exception ( 106A(C)(2)) so long as the pieces were not altered, modified, or destroyed in the process. The court was dealing not just with VARA, but also with the Massachusetts Art Preservation Act. There was no preemption. a) Site-Specific art is a form of integrated art in which the site becomes part of the work. The TOTALITY of the work includes the site (e.g. bldg, land, etc thats owned by other property owners)

3) Issue: Whether VARA applies to integrated or "site-specific art." 4) Holding: The First Circuit held that removal of Phillips' site-specific sculptures was permissible under VARA. 5) Analysis: VARA prohibits any intentional distortion, mutilation, or modification of a work that is prejudicial to an artists honor or reputation. There is a public presentation exception when the modification is the result of conservation, or of the public presentation, including lighting and placement, of the work. The First Circuit determined that VARA could not both apply to site-specific art and also allow for its destruction under the exception. Court reasoned that VARA would either apply to site-specific art, and therefore protect it, or not apply to site-specific work. The court concluded that the public presentation exception applies only to works of art that may be moved without being destroyed. VARA would not apply here to Phillips' site-specific art. Kelley v Chicago Park District (see Notebook 2, p. 9) Holding Right of Integrity: Dist. Ct. did not err in finding that plaintiff-artist had failed to establish violation of his right of integrity in artist display under Visual Artists Rights Act (VARA) where defendant-Park District significantly altered plaintiff's wildflower display in Chicago's Grant Park, even though plaintiff alleged that said act by defendant sufficiently altered his display and damaged his reputation. While plaintiff argued that instant wildflower display was protected as painting or sculpture under VARA, Ct. of Appeals held that said garden display, which was inherently changeable through natural forces, was neither authored nor fixed in senses required for copyright protection, and thus could not qualify for moral rights protection under VARA. Holding Breach of Contract: However, Dist. Ct. erred in finding that plaintiff had established breach of contract claim based on oral commitment made by one Park Bd. Commissioner since individual Commissioner could not unilaterally bind Bd. to any contract under 70 ILCS 1205/4-6 where, as here, there was no evidence that Bd. had given said Commissioner express approval to contract with plaintiff.

Chapter 7. Exclusive Rights and Their Limitations

39/55

Chapter 8. Infringement Actions


To infringe a , the defendant must: 1) a) Reproduce, Adapt, Publicly Distribute, Publicly Perform, and/or Publicly Display the copyrighted work, OR b) violate the digital audio transmission right in a sound recording or the attribution or integrity right in a work of visual art, and 2) Do so without either a) the copyright owners authorization or b) the benefit of one of the limitations in 107-122. 3) Significance: The cases in this chapter illustrate that there are many ways to infringe the copyright in traditional copyrighted works without copying them word-for-word in their entirety

Proving The Claim


Formulating a General Test for Infringement
One standard: Did the def have access to the ptfs work? If so, is the defs work substantially similar to the ptfs work? the casebook authors dont prefer this approach The Arnstein standard (pp. 626-27) Casebook authors think this is the better general standard 1) Does the defs work evidence copying of the ptfs, and if so, 2) Does the copying amount to an improper appropriation? Arnstein Analysis Prong 1 Copying a) a) Almost always proved by indirect (i.e. circumstantial evidence) Def may still escape liability if the ptf fails to show that the taking was impermissible as to kind and amount. Prong 2 Improper Appropriation (includes substantial similarity)

b) Ptf must also show that the intended audience for the two works would find the defs work to be substantially similar

Improper Appropriation Approaches for Determining (see CB p. 644)


Two things to prove: 1) The def appropriated a copyrighted expression, and 2) ptf must prove that the intended will recognize susbstantial similarities between the two works. Verbatim and Pattern Similarities (p. 644) Infringement and De Minimis Use (p. 645) The Subtractive and Totality Approaches (p. 647) Subtractive determine which elements are able and which are not. After subtracting out the non- parts, compare the able parts e.g. Remove Scenes a faire, clich language, etc. Totality Sid & Marty Krofft TV v. McDonalds (p. 649) Totality approach Used the total concept & feel test

Chapter 8. Infringement Actions

40/55

The application of the total concept and feel test to childrens works continued with the seminal case, Sid & Marty Krofft Television Productions v. McDonalds Corp. In Krofft, the plaintiff alleged that the defendants television commercials infringed upon their H.R. Pufnstuf childrens television program, which involved a fantasyland populated by fanciful creatures. The Ninth Circuit established an intrinsic test for substantial similarity that depends solely on the response of the ordinary reasonable person. The court concluded that defendants commercials captured the total concept and feel of the Pufnstuf show.

Copying
Bright Tunes Music Corp. v. Harrisongs Music, Ltd (SDNY 1976) 1) Facts: Bright Tunes (P) sued Harrison (D), claiming that his song "My Sweet Lord" was plagiarized from an earlier song by the Chiffons, titled "He's So Fine," to which P owned the copyright. "HSF" consisted essentially of 4 repetitions of a short musical phrase, followed by four short repetitions of another musical phrase. The second series includes a grace note. "MSL" uses the same series of notes, the first repeated 4 times, and the second repeated 3 times, with an identical grace note in the 2nd repetition. The harmonies of both songs were identical. Testimony revealed that D first began composing the song in London during a press conference. Harrison and the members of the group continued to work on the song, eventually recording it in London. The song was recorded and the music reduced to paper for U.S. Copyright Registration Issue: Is a person liable for copyright infringement even if he infringes a copyrighted work subconsciously? (i.e. if he just sort of recalled the song pattern unintentionally, without purposefully copying the song?) Holding: Yes. Rationale: The critial issues for infringement are 1) whether the works are substantially identical and 2) whether the infringer had access to the original work. Intent to infringe is not required. Here, D did not intentionally infringe. However, the two songs at issue are the essentially the same song with different lyrics. D had access to "He's So Fine." Therefore, D's song constituted copyright infringement. See Ravens Note (p. 632) Can counteract weak access evidence with strong similarity evidence

2) 3) 4)

5)

Ty, Inc. v. GMA Accessories, Inc. (7 Cir 1997) 1) Facts: Ty (P) started manufacturing "Beanie Babies", including a stuffed pig named "Squealer," in 1993. The popularity of the line induced GMA (D) to bring out its own line of bean-bag stuffed animals three years later. D's stuffed pig, Preston the Pig, looks almost identical to Ty's stuffed pig, Squealer. P contended that D's pig was a copy of P's copyrighted pig. Procedure: P obtained a preliminary injunction under the Copyright Act against the sale of Preston the Pig by D. GMA (D) appealed. Issue: May access and copying be inferred when two works are strikingly similar to each other and not to anything else in the public domain? Holding: Yes. Access and copying may be inferred when two works are so similar to each other and not to anything in the public domain, that it is likely that the creator of the second work copied the first. But the inference can be rebutted by disproving access or otherwise by showing independent creation. (Affirmed dist ct).

th

2) 3) 4)

5) Rule: The issue of copying can be broken down into two subissues: 1) whether the alleged copier had access to the work that he is claimed to have copied; 2) whether, if so, he used his access to copy. A similarity that is so close as to be highly unlikely to have been an accident of independent creation is evidence of access. 6) Rationale: D's pig is strikingly similar to P's pig, but not to anything else in the public domain (such as a real pig). D has also not pointed to any fictional pig in the pub dom that Preston resembles. Preston resembles only Squealer. The affidavit of D's designer attesting that she never looked at a Squealer before submitting her design of Preston is only weak evidence of independent creation. The absence of any evidence of how the designer's drawing was translated into the Squealer-resembling production model of Preston, combined with the similarity of that model to Squealer (and

Chapter 8. Infringement Actions

41/55

to nothing in the public domain) overbore the weak evidence of the affidavit. There was no error of law, no clear error of fact, and no abuse of discretion in granting the preliminary injunction.

Improper Appropriation
Fill in pp. 644 650 (see notebook) Nichols v. Univsersal Pictures Corp. (2d Cir 1930) 1) Facts: Nichols (P) authored and copyrighted the play Abies Irish Rose. The work depicted the marriage between a Jewish boy and an Irish Catholic girl, their deception of their religious fathers, and the eventual acceptance and reconciliation. Universal Pictures (D) produced The Cohens and the Kellys, a movie about the marriage between an Irish boy and a Jewish girl. While not emphasizing the religious, the movie centered on the interactions of the two families. Procedure: Nichols (P) brought suit for infringement. The district court dismissed because the claim was on common themes. Issue: Is copyright protection of literary property limited merely to protecting the literal text of the work? Holding: No, copyright protection of a literary work cannot be limited to the literal text; otherwise, a plagiarist could escape liability with immaterial changes. However, protection cannot extend to the ideas of the copyrighted work. It can only express the expression of the work. (Affirmed dist ct) Rule: Ideas cannot be copyrighted. Only the expressions of those ideas can be copyrighted. Rationale: Court said although two pieces may correspond in some ways, the similarities were not enough to constitute infringement, as the focus of the stories were quite different (religion v. money). Abies Irish Rose focuses on religious zeal the father doesnt want the son to marry outside the religion. In the Cohens and the Kellys, the focus was on money (see CB p. 303). Also, the character portrayal differed.. The stories themselves are different. Every work can be abstracted on several levels, ranging from general statements of what the work is about, to the specific reproduction of the work. (e.g. Title (general) Theme Setting Characters Plot Dialogue (specific)). Between this series of abstractions lies the boundary between protectable expression and unprotectable ideas. The only matter common to the two works is a quarrel between a Jewish and an Irish father, the marriage of their children, the birth of grandchildren, and reconciliation. This is too generalized an abstraction from what Nichols (P) wrote, and thus was only a part of her ideas.

2) 3) 4)

5) 6)

The ordinary observer test


Peter Pan Fabrics, Inc. v. Martin Weiner Corp. (2d Cir 1960) 1) 2) 3) 4) 5) 6) Facts: Both the ptfs and def were converters of textiles used in the manufacture of womens dresses. Converters buy uncolored cloth, imprint the cloth with ornamental designs, and then sell the cloth to dressmakers. Procedure: The def appealed an injunction issued by the trial court to prevent it from copying one of the plaintiffs designs. Issue: In determining whether the copyright in a design has been infringed, is the proper test whether an ordinary observer would regard the infringing and non-infringing works to be aesthetically similar? Holding: Affirmed Rule: The test for determining copyright infringement in design is the ordinary observer test. Rationale: Both designs have the same general color. The arches, scrolls, rows of symbols, etc. from one resemble those on the other, but are not identical. Moreover, the patterns in which these figures are distributed to make up the design as a whole are not identical. However, the ordinary observer, unless he set out to detect the differences, would probably overlook them, and think the two designs are aesthetically the same.

7) Note 8, p. 658 8) Note 10, p. 659 -

Chapter 8. Infringement Actions

42/55

The More Discerning Observer test


Laureyssens v. Idea Group, Inc. (2d Cir 1992) 1) Facts: Lareyssens (P), designed toys and puzzles. By 1991, he filed certificates of copyright registration for six of his designs. The certificates stated that the nature of authorship consisted of the shape of the pieces. His Happy Cube puzzle consisted of pieces that could be assembled both flat and in cube form. In 1990, P discovered that Idea (D) was manufacturing and selling an identical puzzle called Snafooz. Although D conceded that it had copied Snafooz from Happy Cube and that it needed Ps permission to market Snafooz in the US, the parties failed to reach an agreement. D attempted to develop its own version of the puzzles. D hired Brewer, who stated that he developed the puzzle on his own, using the old Snafooz puzzles as a guide. The Snafooz puzzle used a 6 notch-width puzzle piece, whereas Happy Cube used a 5 notch-width puzzle piece. Procedure: P brought suit for copyright and trade dress infringement and unfair competition. The court granted a preliminary injunction on the trade dress and unfair competition claims. The court denied a preliminary injunction on the copyright claim. Idea (D) appealed and P cross-appealed from the denial on the copyright claim. Issue: In determining whether unlawful appropriation exists in a copyright infringement case involving a work that contains both protectable and nonprotectable elements, must it appear to the ordinary observer that the works are aesthetically the same only with respect to the protectable material? Holding; Yes. Grant of the preliminary injunction on the trade dress infringement and unfair competition is reversed. Denial of the preliminary injunction as to the copyright infringement claim is affirmed. Rule: In order to establish a claim for copyright infringement, the plaintiff must prove 1) ownership of a valid copyright and 2) unlawful copying. It must appear to the ordinary observer that the works are aesthetically the same ONLY WITH RESPECT TO THE PROTECTABLE MATERIAL. Substantial Similarity Test: . The burden is on the plaintiff to show access and substantial similarity. The plaintiff must first show that the copyrighted work was actually copied, through either direct or indirect evidence. Indirect evidence includes access to the copyrighted work, similarities that demonstrate copying, and expert testimony. If the ptf demonstrates actual copying, he must next demonstrate that such copying constitutes an improper appropriation by showing substantial similarity as to the protected material exists between the two works. Unlawful Appropriation Test: The test for UA is whether substantial similarity as to protected material exists between the two works that are the subject matter of the litigation. The standard is whether an ordinary observer would regard the works as aesthetically the same (unless he set out to determine the differences). When a work contains both protectable and unprotectable material, the inspection must be more discerning and ignore the elements that are not copyrightable.

2)

3)

4) 5)

6)

7)

8) Rationale: Here, P had a valid copyright in the puzzle designs. Actionable copying may be inferred from the defs access to the copyrighted work and the substantial similarity between the copyrighted and infringing works. Here, there is evidence of access to the copyrighted work, and there are similarities between the puzzles that raise a question as to actual copying. The issue is whether the district court erred in determining no issue existed as to the unlawful appropriation of the copyrighted material. The dist ct concluded that Ps copyright extended only to the particular expression of the idea of a flat-to-cube puzzle, manifested in the particular shapes of HIS puzzle pieces. The dist ct did not abuse its discretion. An ordinary observer would find the change to a 6 6-notch-width design (Snafooz) results in a qualitatively different puzzle. 9) Note 4, p. 669 10) Note 11, p. 672-673 Fill in Computer Associates International, Inc. v. Altai, Inc. (2d Cir 1992) 1) Facts: Computer Associates (CA) (P) designed, developed, and marketed various types of computer programs, including CA-Scheduler, a job scheduling program containing a subprogram called Adapter. Adapter was a wholly integrated component of CA-Scheduler with no capacity for independent use. Altai (D) began marketing its own job scheduling program, called Zeke. D decided to rewrite Zeke to run in conjunction with a different operating system, and approached Arney, a computer programmer who worked for CA. When Arney left CA to work for Altai (D), he took copies of the source code for two versions of Adapter. No one at Altai knew he had the Adapter codes and was using

Chapter 8. Infringement Actions

43/55

them to design Ds new component-program, Oscar (ver 3.4). Arney copied approx 30% of Oscars Code from Adapter. 2) Procedure: When CA learned that Altai may have appropriated parts of Adapter, it brought this copyright and trade secret misappropriation action against Altai. Altai then initiated a rewrite of Oscar (ver 3.5). Altai shipped only Oscar 3.5 to new customers, and a free 3.5 upgrade to all customers who had previously purchased ver 3.4. Claim 1 Copyright Infringement: The dist ct awarded CA $364,444 in actual damages and apportioned profits for copyright infringement regarding Oscar 3.4. Claim 2 Substantial Similarity: Dist ct denied relief, finding that Oscar 3.5 was not substantially similar to Adapter. State Law Trade Secret Misappropriation Claim: Dist ct held that the state law trade secret misappropriation claim against D was pre-empted by the federal copyright act. On appeal, D conceded liability nd rd for copying Adapter into Oscar 3.4. Thus, only CAs 2 and 3 claims were addressed on appeal. Issue: To warrant a finding of copyright infringement, must the protectable, nonliteral elements of one computer program be substantially similar to those elements in a second program? Holding: Yes. (Affirmed dist ct) Rule: The literal elements of computer programs (e.g. source and object code) are copyrightable. Rationale: Altai made sure the literal elements of its revamped Oscar program were no longer substantially similar to the literal elements of CAs Adapter. If the nonliteral structures of literary works are protected by copyright, then the nonliteral structures of computer programs are also protected by copyright. The three-step approach outlined here (below) not only comports with, but advances the constitutional policies underlying the copyright act. Upon a thorough review of the voluminous record in this case, this court finds no error in the dist ct, let alone clear error. Each program consists of a number of interacting subprograms, which work together to perform the functions purpose. Each of the subprograms may represent one or more separate ideas. 3-step procedure for determining whether the nonliteral structures are protectable: 1) The abstractions test: Retrace and map each of the designers steps in the opposite order in which they were taken during the programs creation. 2) The filtration test: Examine the structural components at each level of abstraction to determine whether their particular inclusion at that level was dictated by considerations of efficiency; required by factors external to the program itself; or take from the public domain, thus making them nonprotectable. 3) Comparison: Once a court has sifted out all those elements of the allegedly infringed program, there may remain a core of protectable expression. At this point, the courts substantial similarity inquiry focuses on whether Altai (D) copied any aspect of this protected expression, as well as an assessment of the copied portions relative importance with respect to CAs (P) overall program. Note on the Merger Doctrine: The court discussed the merger doctrine. The underlying principle of the merger doctrine is: When there is essentially only one way to express an idea, the idea and its expression are inseparable, and copyright is no bar to copying that expression. With computers, this means that when specific instructions are the only and essential means of accomplishing a given task, their later use by another will not be infringement (even if they are copyrighted). a) Note: In Altai, the court rejected the Whelan approach saying it was too simplistic. Altais 3 step approach considers the many ideas that may be embodied in a computer program

3) 4) 5) 6)

7) 8)

9)

10) Notes

b) Note 2, p. 689 - Merger of Computer Program Structure with Underlying Idea: Whelan Associates v. Jaslow Dental Laboratory (3d Cir 1986) c) Facts: The Principal of Ptf company (herself a computer expert) had worked closely w/ defs (data processing rookies running a dental lab) to develop a specialized biz mgmt prog under a profit-sharing contract. Defs then decided to go it alone, marketing a program designed to do the same thing, and they terminated the K.

d) Issue: At the literal code level, the programs were not at all identical. Finding for Ptf would require the ct to find a std that gave legal weight to similarities in file structure, screen outputs, and certain subroutines used in programming, as well as overall structural similarities. e) Holding: Ptf wins. Ct rejected several defenses in particular, the arg that the structure of a piece of software was so integral to its unprotected idea that it merged.

Chapter 8. Infringement Actions

44/55

f)

Significance: Whelan suggested that the scope of Copyright was more extensive than that for other types of works

Tufenkian Import/Export Ventures, Inc. v. Einstein Moomjy, Inc. (2d Cir 2003) (this case starts the Apr 10 reading) This copyright infringement case involves two textile designs, each of which combines, with modifications, the primary border and the half field of two unrelated public domain carpets, one a classical Indian Agra and the other a Persian antique. Viewed uncritically, the two designs at issue are substantially similar. For the defendant's rug to infringe upon the plaintiff's design, however, the defendant's composition must be substantially similar to that which is original in the plaintiff's expression. Procedure: The district court (S.D.N.Y., Pauley, J.) found no infringement, concluding as a matter of law that whatever substantial similarity there may be emerges from unprotected public domain materials in the allegedly infringed design. See Tufenkian Imp./Exp. Ventures, Inc. v. Einstein Moomjy, Inc., 237 F.Supp.2d 376 (S.D.N.Y.2002). We disagree.

Chapter 10. Fair Use and Affirmative Defenses


Fair Use Factors 107 1) 2) 3) The purpose and character of the *defendants+ use (including whether commercial or for non-profit purposes) The nature of the copyrighted work The amount and substantiality of the portion used in relation to the COPYRIGHTED WORK as a whole a) 4) (not to the defendants work) though some courts misapply this prong and look to the substantiality in the defs work

The effect of the use upon the potential market for or value of the copyrighted work

The Fundamentals of Fair Use


Judicial Origins
Folsom v. Marsh (D. Mass 1841) 1) 2) Folsom established the origins of the fair use test. Facts: The ptf alleged that the def copied its copyrighted work, the Writings of President Washington, which consisted of twelve volumes. The defs work, called Life of Washington, consisted of two volumes in which Washington tells the story of his life. The complaint alleged that the defs work inserted various letters from the original work in their entirety. The copied pages constituted roughly 1/3 of the second work, and consist mainly of official and private letters and documents. Issue: In determining whether a use of copyrighted material is justifiable, is it a defense that only a part and not the whole original work was appropriated? Holding: No it is not a defense that only a part of the original work was appropriated. (But what about in the case of music?? Small samples?) Rule on Infringement and Fair Use (remember, this is OLD SCHOOL): Copyright infringement does not require the entire work to be copied, nor does a finding of infringement depend on the particular quantity taken. The court must consider 1) the value of the appropriated materials, 2) the importance of such materials to the sale of the original work, 3) the nature of the portion taken, 4) the quantity and value of those materials, and 5) the extent to which the taking may effect the sales or profits of the original work. Rationale: Here, the defs appropriated 319 letters to which the ptf had the exclusive copyright (extent of taking). The defs work was based primarily on these letters, which were reprinted in their entirety (importance of taking to defs work). This clearly infringed ptfs copyright.

3) 4) 5)

6)

Chapter 10. Fair Use and Affirmative Defenses

45/55

7) Note 3, p. 817 Sony v. Universal City Studios (see below p. 50)

Analyzing Fair Use


Campbell v. Acuff-Rose Music, Inc (U.S. 1994) 1) Facts: In 1964, Roy Orbison and William Dees wrote the song, Oh, Pretty Woman and assigned their rights in the work to Acuff (P). P registered the song for protection. In 1989, the rap group, 2 Live Crew (D), wrote a song called Pretty Woman and informed P that it was a parody of the Orbison song. Accordingly, D would give credit and acknowledge Ps ownership of the original, and were willing to pay a fee for the use. P refused to grant permission to use the song. In 1989, D released the song and gave credit to Orbison and Dees as the authors and Acuff (P) as the publisher. Procedure: Acuff sued 2 Live Crew (D) and its label for infringement. The dist court granted summary judgment for D, finding that its version was a parody under the affirmative defense of fair use. The court of appeals reversed and remanded. SCOTUS granted cert. Issue: Does parody constitute fair use under 107 of the 1976 Copyright Act? Holding: Yes. Parody constitutes fair use under 107. The court of appeals erred in finding that the commercial nature of the parody made it presumptively unfair use. Reversed and remanded. Fair Use Test: 1) The purpose/character of the use, including whether it is of commercial nature or nonprofit, educational purposes; 2) The nature of the plaintiffs work; 3) The amount and substantiality of the portion taken; and 4) the effect of the use on the potential market value of the copyrighted work (including the original and any derivative works). Rationale: 1) Parody has transformative value and, similar to comment or cricism, may claim fair use under 107. The threshold question is: whether a parodic character may reasonably be perceived. Here, 2 Live Crews version could be reasonably perceived as commenting upon or criticizing the original work. The ct of appeals erroneously confined its consideration of the first factor to the commercial nature of the use, stating that the presumption that any commercial use of copyrighted material is unfair. The commercial use is only one factor to be considered. The broader question is the purpose and character of the use. 2) Not analyzed??. 3) The court must analyze the amount use AND ALSO its importance to the COPYRIGHTED work (not to the defendants work, but the plaintiffs). Here, 2 Live Crew copied the th first line of the original work, but then lyrically, it departed substantially. 4) The 4 factor includes harm to the market for the original as well as any derivative works. The ct of appeals erroneously presumed that since the use of the original was commercial, a likelihood of future harm to P existed. However, neither D nor P introduced any evidence of the effect of the parady on the market for potential rap derivative versions of the original. Takeaways: The presumption that any commercial use of copyrighted material is unfair is WRONG

2)

3) 4) 5)

6)

7)

8) Note 12, p. 821 Disney v Air Pirates 9) Note 9, p. 819 Dr. Seuss Enterprises v. Penguin Books 10) Note 9, p. 820 Leibovitz v. Paramount Pictures 11) Note 13, p. 822 Suntrust Bank v. Houghton Mifflin Co (11 Cir 2001) 12) Suntrust Bank v. Houghton Mifflin Co. (11 Cir 2001) 13) Facts: Houghton Mifflin has rights to a book called The Wind Done Gone (TWDG), a critique/parody of Gone with the Wind (GWTW). Gone with the Wind is one of the worlds best-selling books. SunTrust Bank is the trustee of the Michelle Trust that holds the copyrights to that book. TWDG used several characters, plots, and major scenes from GWTW. 14) Holding: TWDG is fair use it was a Parody that REQUIRES the original work (cant stand on its own). Though P has a valid copyright, Ds use is fair use, and not an infringement 15) Rule: Factors to Determine Fair Use: Fair use is not copyright infringement o 1) The purpose/character of the use, including whether such use is of a commercial nature or is for nonprofit educational purposes;
th th

Chapter 10. Fair Use and Affirmative Defenses

46/55

o o o o

Purpose educational is more fair than commercial Character more transformative is more fair and creative than less transformative 2) The nature of the copyrighted work; 3) The amount and substantiality of the portion used in relation to the copyrighted work as a whole; AND 4) The effect of the use upon the potential market for or value of the copyrighted work Evidence of harm to the market or value of the copyrighted work weighs against the infringer Notes Factors 1 and 4 are major

16) Notes: 17) Parody as fair use: The parody branch of fair use doctrine is a means of fostering the creativity that the copyright law protects

Conceptual Issues In Fair Use


Copyright and the First Amendment
Harper & Row, Publishers, Inc. v. Nation Enterprises (U.S. 1985) Facts: Harper & Row (P) obtained the rights to publish President Fords memoirs, A Time to Heal: The Autobiography of Gerald R. Ford. Time magazine contracted for the rights to preview the work immediately prior to publication. Prior to the publication of the article by Time magazine, Nation Enterprises (D), publisher of The Nation magazine, obtained a copy of the Ford manuscript. The Nation published an article that quoted 300 words (verbatim) of the manuscript regarding the Nixon pardon. Time then declined to run the article it had planned and canceled its contract with P. Procedure: P sued Nation Enterprises (D) for infringement. The dist ct awarded damages for infringement. The 2d Cir. reversed, holding Nations use to be fair use under 107. SCOTUS granted cert. Issue: Is publication of portions of a work soon to be published a fair use? Holding: No. Prior publication of a work awaiting publication is not fair use. Rationale: The notion behind the fair use doctrine is that one using a copyrighted work should not have to obtain a copyright holders permission to use the copyrighted work in a situation where a reasonable copyright holder would in fact grant permission. For one, it is not reasonable to expect a copyright holder to allow another person to scoop it by publishing his material ahead of time. Secondly, this case is not fair use on analysis of the 4 factors. Factor 1: Defs use was commercial and non-transformative (weighs in favor of Ptf), and also done in bad faith. Factor 2: Ptfs work was an autobiographyfactual info. Factor 3: The def used 300-400 of about 200,000 words of Ptfs work. The court INCORRECTLY FOCUSED ON HOW MUCH THE USE DEFS work. It determined that the use was critical to defs work. This is the courts manipulation of the rule to get the judgment it wants. Factor 4: Substantial market harm Because of the harm, Time canceled the contract & refused to pay license fee. Also, defs publication pre-emption cuts into moral rights (first publication). The court point out that the two fair use factors ( 107) most salient here are 1) purpose of use and 2) effect on the market. Normally, a fair use will not be one of economic competition with the copyright holder, which is precisely what prior publication of a copyrighted work is. Further, the effect on the market of such a use greatly lessens the market value of the copyrighted work.

The Meaning of Transformative Use


Bill Graham Archives v. Dorling Kindersley Ltd. (2d Cir. 2006) Facts: Plainitff owned the coprights to several Grateful Dead posters and concert tickets. Defendants published reduced size images of these items in a 480 page biography of the Grateful Dead. Holding: The Second Circuit upheld the trial court decision that the use of plaintiffs images was Fair Use authorized by the Copyright laws of the United States.

Chapter 10. Fair Use and Affirmative Defenses

47/55

Analysis: 1) Defendants use was commercial, but highly transformative (from artistic to biographical use) weighed in favor of def; 2) 2) Ptfs work was creative & highly protected. The ct said this factor doesnt weigh heavily because most works are highly protected could even be considered neutral; 3) Amount/Substantiality of copying = wholesale copying; but this was ok because in this case, the whole images were necessary for the biography . Key language = the defs use was tailored to further the transformative use weighed in favor of def 4) No market harm Ptf cannot speculate on its future licensing. Ptf cannot use the basis of the lost licensing fee to the defendant as the whole argument. The ptf cannot pre-empt transformative markets cannot prevent the def from entering fair use markets See note 5, p. 847 Rogers v. Koons and Blanch v. Koons (more fair use cases)

Actual and Potential Market Effect


pp. 853-856

Fair Use and the Internet


Kelly v. Arriba Soft Corp. (9th Cir. 2003) 1) Facts: Kelly (P) is a photographer with copyrighted images. Arriba (D), a search engine, copied some of these images and displayed them on its website. D produces its list of search results as small pictures rather than in text. Full-sized copies of images were downloaded onto Ds servers, and then smaller thumbnails were made. Some of the thumbnails rd linked directly to his site; others linked to 3 party sites that had his images. At first, when a user clicked on a thumbnail, the original full-sized image was imported directly from the originating website along with text, a link, and Arriba advertising. Later, when the thumbnail was linked, two windows appeared with one containing a full-sized image and the other displaying the originating website. D copied 35 of Ps photos. Procedure: P sued for infringement. Dist ct granted Ds motion for summary judgment, finding that Ds reproduction and display of Ps images constituted a non-infringing fair use. The character and purpose of Ds use was significantly transformative and did not harm the market for, or value of, Ps works. P appealed. Issue: (1) Was Ds use of Ps images as thumbnails in its search engine a fair use? (2) Did Ds inline linking to and framing of Ps full-sized images infringe upon Ps exclusive right to display the d works publicly? Holding: (1) Yes Arribas (D) use of Ps images as thumbnails was fair use. (2) Yes, Arribas (D) inline linking/framing of Ps full-sized images infringed on Ps exclusive right of public display. (Affirmed as to thumbnails, reversed as to fullsized display) Rationale: (1) Fair use 4-factor analysis: 1. Nature of Defs use: Although D operates its website for commercial purposes, its use of Ps images was more incidental and less exploitative in nature than more traditional types of commercial use. Ds use was also transformative because the thumbnails were much smaller, lower-resolution images that served an entirely different purpose than Ps original images. i.e. Ds use did not supersede Ps use, but transformed it. Nature of Ps work: Though Ps works were creative (and highly protected), they had been on the internet before D copied them. Amount & Substantiality Taken: D copied Ps entire images. But, it was necessary for D to use the whole images so that users could recognize the images and decide whether to click through. Market Harm: Ds use did not harm the market for P or the value of his images. Users of Ds site would be guided to Ps site, and the thumbnails would not be a substitute for the full-sized images,

2)

3) 4)

5)

2. 3. 4.

Chapter 10. Fair Use and Affirmative Defenses

48/55

because when the thumbnails are enlarged, they lose their clarity. Ps ability to sell or license his fullsized images is not harmed. (2) Public Display Analysis: 1. 2. Ds inline linking/framing displayed Ps work to the public (via website) without Ps permission. Thus, when D imports Ps images into its own web page, D is infringing upon Ps public display rights. Transformative Use: Furthermore, the display of the full-sized images was not transformative because it did not enhance Ds search engine, but rather, had the same purpose as Ps purpose (to display the images), and added no expression to the images. Market Harm: By linking to the full-sized images, D harmed Ps markets because people would be deterred from going to Ps site for the images.

3.

Fair Use and Technological Protection Measures under the DMCA


Notes pp. 882-885

De Minimis Use
Three aspects (to be considered separately) 1) A technical violation of a right so trivial that the law will not impose legal consequences, (e.g. a copyrighted sculpture appears in a photograph you take of someone while on vacation), OR 2) Quantitatively Trivial Amount Taken (e.g. just not that much) 3) Qualitatively Trivial Amount (an insignificant portion; e.g. taking the layout/style of an acknowledgements section from a ed book for your own book Parody (see Suntrust v Houghton Mifflin)

Chapter 9 Secondary Liability


Dont worry about the fact that Chapter 9 comes after Chapter 10 in this outline. Mind your business. Thats the order we followed in class

Contributory Infringement and Vicarious Liability


Notes pp. 728-729

Case Law
Fonovisa, Inc. v. Cherry Auction, Inc. (9th Cir. 1996) 1) Facts: Cherry Auction (D) operated a swap meet in Fresno, CA. D collected a daily rental fee from each vendor and supplied parking and advertising. It also retained the right to exclude vendors for patent and trademark infringement. Nevertheless, it allowed vendors to openly sell counterfeit recordings of Latin music copyrighted by Fonovisa (P). It also refused to assist an ongoing sheriffs investigation into the sale of counterfeit products. Procedure: P sued D for contributory copyright infringement, vicarious copyright infringement, and contributory trademark infringement. The dist ct dismissed the suit because it concluded that D neither supervised nor profited from the vendors sales. Issues: (1) May one be vicariously liable for infringement if A) one has the right and ability to supervise the infringing activity and B) one also has a direct financial interest in such activities? (2) May one who, with knowledge of the infringing activity, induces, causes, or materially contributes to the infringing conduct of another, be held liable as a contributory infringer?

2)

3)

Chapter 9 Secondary Liability

49/55

4) 5)

Holding: (1) Yes (D is vicariously liable). (2). Yes (D is liable under contributory infringement) Rule: (1) Vicarious Infringement (Rooted in Agency/Respondeat Superior): One may be vicariously liable for infringement if A) one has the right and ability to supervise the infringing activity and B) one also has a direct financial interest in such activities. (2) Contributory Infringement (Rooted in Tort Law): One who, with knowledge of infringing activity, induces, causes, or materially contributes to the infringing conduct of another may be held liable as a contributory infringer. Rationale: Here, D protected the infringers identities when it refused to share basic information about them with the local sheriff. Moreover, providing the site and facilities for known infringing activity is sufficient to establish contributory liability. Lastly, since D clearly disregarded its vendors blatant trademark infringements, P state a claim for contributory trademark infringement.

6)

Manufacturing and Distributing Copying Devices


Notes p 736-737 Case Law Sony Corp. of America v. Universal City Studios, Inc. (U.S. 1984) Time-Shifting and the The Staple Article of Commerce Doctrine 1) 2) Facts: Sony (D) manufactures and sells home video recorders (VCRs). Universal (P) held the copyrights on some of the shows telecast to the public on TV, and admittedly, videotaped by some of the public that had bought VCRs. Procedure: P sued D for infringement. Trial court denied the money damages, equitable accounting of profits, and injunction against the manufacturing and selling of VCRs sought by P. But the 9th Cir reversed and held D liable for contributory infringement. 9th Cir ordered dist ct to fashion appropriate relief. The evidence showed that the primary use of VCRs for most owners was time-shifting, i.e. the practice of recording a program to view it once at a later time, and then erasing it (which would favor Sonys fair use defense). However, a substantial number of those surveyed said they had accumulated libraries of tapes of programs they had recorded (which would favor Universals prosecution). Sony (D) introduced evidence that a substantial number of copyright owners (e.g. TV shows) on programs did not object to home time-shifting because it increased their audience (this favored Sonys fair use defense). Issue: Does the use of home VCRs for home time-shifting of televised programs constitute of fair use of copyrighted material? Holding: Yes fair use (reversed). Rationale: Under the staple article of commerce doctrine, the sale of copying equipment, like the sale of other articles of commerce, does not constitute contributory infringement if the product is widely used for legitimate, unobjectionable purposes. Thus, there is not contributory infringement here. Dissent: Market Harm: There is a wide market of people who wish to view programs at times other than the original broadcast time. Sonys machines potentially deprive the copyright holders of the ability to exploit this market, which means time-shifting cannot be deemed a fair use. Productive Use: Fair use was designed to protect only uses that are productive. There is no prior case in which the reproduction of a copyrighted work for the sole benefit of the user has been held to be fair use. (I guess this is a first)

3) 4) 5)

6)

Secondary Liability on the Internet


Introduction
Notes p. 750-753

Chapter 9 Secondary Liability

50/55

Limitation of Liability for Online Service Providers


pp 753-759

Peer-to-Peer File Sharing


pp 759-760 Metro-Goldwyn-Mayer Studios, Inc. v. Grokster, Ltd. (U.S. 2005) 1) Facts/Procedure: Grokster and other companies distributed free software that allowed computer users to share electronic files through peer-to-peer networks. In such networks, users can share digital files directly between their computers, without the use of a central server. Users employed the software primarily to download copyrighted files, file-sharing which the software companies knew about and encouraged. The companies profited from advertising revenue, since they streamed ads to the software users. A group of movie studios and other copyright holders sued and alleged that Grokster and the other companies violated the Copyright Act by intentionally distributing software to enable users to infringe copyrighted works. The district court ruled for Grokster, reasoning that the software distribution companies were not liable for copyright violations stemming from their software, which could have been used lawfully. The Ninth Circuit affirmed.

12) Holding: Supreme Court found that since they were pretty much advertising that their system is for infringing, and even encouraging it, they are contributing to the infringement and thus liable. 13) Concurrence Breyer: Breyer concurring takes up Sony question left open by the ct: whether defs product is capable of substantial or commercially significant non-infringing uses. a) On the side that the appellate court was correct to grant summary judgment to def that their product IS capable of substantial or commercially significant non-infringing uses.

u) That def passes the Sony test because (1) at least 10% is non-infringing (similar to Sony which 9% was noninfringing there) (2) defs system, if 10% is the bottom foundation, is capable to be used legitimately over time (looking to future). v) Breyer is NOT saying that def should not be liable or that Sony protects def (quite the opposite), what he is saying is that the Sony standard seeks to protect the development of [that] technology [(P2P)] more generally [so that it can get to a point where it is used for non-infringing purposes].

w) Breyer says Sony itself sought to strike a balance between a copyright holders legitimate demand for effective (not just symbolic) protection and the rights of others to freely engage in substantially unrelated areas of commerce. x) Sony provides entrepreneurs with needed assurance that they will be shielded from copyright liability as they bring valuable new technologies to the market, allowing development without worrying re liability. Deters those who distribute products that have no real function but just to infringe. Sony rule IS supposed to be hard to put liability on those tech dev unless their product was designed to infringe. And that Sony is forward looking and applies to future tech. That which tech or which way to do things is not necessarily up to the judges through Sony unless blatant and thus thats why there has been so few contrib. or vicarious infringement. To modify Sony to require more evidence burden on the def would crush what Sony set out-let tech develop and require more evidence would bring uncertainty as to what is enough and other costs

14) Dist Ct - NOT CONTRIBUTORILY LIABLE FOR COPYRIGHT INFRINGEMENT BY USERS OF ITS P2P FILE SHARING PROGRAM 15) One who distributes a device with the object of promoting its use to infringe copyright (as shown by clear expression or other affirmative steps taken to foster infringement) is liable for the resulting acts of infringement by third parties 16) The issue: distribution of a product that is capable of lawful and unlawful uses y) P2P Networks iv) No central computer server to mediate the exchange of information or files

Chapter 9 Secondary Liability

51/55

17) Is the distributor liable for the action of 3rd parties infringement? 18) There were efforts by to encourage illegal downloading 19) Supreme Court z) Focused on the definition of "commercially significant non infringing uses" v) Where an article is good for nothing else but infringement, there is no legitimate public interest in its unlicensed availability and there is no injustice in presuming or imputing an intent to infringe

aa) The doctrine also absolves the equivocal conduct of selling an item with substantial lawful as well as unlawful uses, and limits liability to instances of more acute fault than the mere understanding that some of ones products will be misused vi) This leaves breathing room for innovation and rigorous commerce bb) Did the D's product provide for commercially significant non-infringing uses? cc) Was there intent of promoting infringement vii) Where evidence goes beyond a products characteristics or the knowledge that it may be put to infringing uses and shows statements or actions directed to promoting infringement, Sonys Staple Argument will not preclude liability

dd) The court found intent and ruled that one who distributes a device with the object of promoting its use to infringe copyrights is liable for the resulting acts of the infringement by third parties viii) ix) Evidence of active steps such as advertising an infringing use or instructing how to engage in an infringing use show an affirmative intent that the product be used to infringe And a showing that infringement was encouraged overcomes the laws reluctance to find liability when a defendant merely sells a commercial product suitable for some lawful use There is a definite tendency to impose greater responsibility upon a defendant whose conduct was intended to do harm or was morally wrong

ee) Inducement Rule x)

ff) Balancing act: Purposeful culpable expression and conduct that does not jeopardize legitimate commerce or discourage innovation that has a legitimate application

Other Online Issues


pp 772-774

Technological Protection Measures and Circumvention Devices


Anti-Circumvention Measures
See Notebook 2, p. 27 CB p. 774 Act 1201

Chapter 9 Secondary Liability

52/55

Chapter 11. Remedies, Pre-Emption and Related Bodies of Law


Remedies Under Federal Law
pp 899-901

Introduction
pp. 901-903

Non-Monetary Relief
pp. 903-904

Preliminary and Permanent Injunctions


Paramount

Impoundment and Disposition


pp 913-914

Idea Protection
Murray v. National Broadcasting Co., Inc (2d Cir. 1988) Facts: Murray (P), an employee of NBC Sports, submitted five written proposals for future TV shows to another division of NBC (D). Murray allegedly informed (D) that if it were interested in any of the proposals, he expected to be named executive producer and to receive appropriate credit and compensation as creator of the eventual program. He also told NBC the ideas were being submitted in confidence. NBC requested that Murray "flesh out" his proposal for a series named "Father's Day." Murray outlined a series like the old "Dick Van Dyke Show" about a middle-class family featuring a father, working spouse, and five children. However, in Father's Day, the cast would be all black, and the show would feature blacks in non-stereotypical ways (e.g. father = lawyer, mother = doctor, they live in NY brownstone). P suggested Bill Cosby for the father. Fifteen years earlier, Cosby himself had suggested a "dream series" like the "Dick Van Dyke Show" that would demonstrate that there were no differences between black and white families. NBC rejected Murray's proposal, but four yearrs later premiered "The Cosby Show," in which Bill Cosby played a doctor, his wife was a lawyer, and they had 5 children--a black upper-middle class sitcom. The series had top ratings. Procedure: P sued D for idea misappropriation, race discrimination, and false designation of origin under the Lanham Act. The dist ct granted NBC (D) summ judg on the grounds that P's idea was not original with him. P appealed. Issue: If one submits an idea to another that is not novel, may a promise to pay for its use be implied or an asserted agreement for it be enforced? Holding: No. There is no infringement here. (Affirmed) Rule: Novelty and Originality are required. i.e., When one submits an idea to another, no promise to pay for its use may be implied, and no asserted agreement enforced, if the elements of novelty and originality are absent. Rationale: Non-novel or non-unique ideas do not constitute property, are in the public domain, and do not give rise to claims of unauthorized use. Here, the non-stereotypical portrayal of a black middle-class family in a sitcom are not novel; P's idea essentially consisted of nothing more than a variation of a basic theme, an adaptation of existing knowledge, and of known ingredients. All of P's claims (i.e. misappropriation, fraud, race discrimination under 42 U.S.C. 1981-82, and

Chapter 11. Remedies, Pre-Emption and Related Bodies of Law 53/55

Lanham Act 43(a)) require that he have a property interest in his idea. But there can be no contractual or property right, or right of attribution, in a non-novel idea. Dissent: Summary judgment was incorrect. D admitted its agreement with the producers of "The Cosby Show" was "unique intellectual property" and that the "property" was the "program idea, and/or program format which forms the basis" for the TV show. Further, D admitted that P had rights in his idea, and both Bill Cosby and D's president state that "The Cosby Show" was novel and unique. Because there is evidence that P's idea was novel, the question of novelty is one of fact, and summary judgment was granted prematurely.

Damages
Plaintiffs Damages and Defendants Profits
Polar Bear Productions, Inc. v. Timex Corporation (9th Cir. 2004) See Notebook 2, p. 32 CB p. 621 Salinger Case Notebook 2 p. 32 eBay case NB 2, p. 32

Statutory or In Lieu Damages


Columbia Pictures Television v. Krypton Broadcasting of Birmingham, Inc. (9th Cir. 1997) ( 1) 2) Facts: Krypton (D) owned 3 TV stations in the Southeast. Columbia (P) licensed several TV shows to the stations. After the stations were delinquent in paying royalties, Columbia sued. Procedure: During suit, Columbia dropped all of its claims except its copyright claim against Krypton's (D) owner, Feltner (D). The dist ct found Feltner (D) vicariously and contributorily liable for copyright infringement. The court awarded Columbia (P) summ judg on liability, and after a bench trial, Columbia was awarded $8.8M in damages and $750K in atty fees & costs. Feltner and Krypton (Ds) appealed. Issue: May statutory damages be awarded under 504(c)(1) for every infringement involved in the suit in respect to a single work, or for which two or more defs are found to be jointly and severally liable? Holding: Yes. Statutory damages may be awarded under 504(c)(1) for A) every infringement involved in the suit in respect to a single work, or B) for which two or more defs are found to be jointly and severally liable. Rule: See 504(c)(1). The 1976 Act allows a copyright owner to elect to receive statutory damages in lieu of actual damages under 504(c)(1). Under this section, upon a finding of infringement, the ptf is to receive an award of not less than $500 or more than $20,000 (up to the court's discretion). If the court finds a willful infringement, it may increase the damages award to no more than $100,000. If the court concludes it was an innocent infringement, the award may be reduced to $200. "Willful" is defined as knowingly infringing another's copyright.

3) 4) 5)

6) Rationale: Here, Columbia elected statutory damages. The court found Feltner's violation was willful and assessed statutory damages at $20,000 per violation. In order to rebut a finding of willfulness, the def must show a good faith and reasonable belief that his conduct was not infringing. The evidence of 440 TV shows being broadcast after Columbia's termination of its license agreements and 415 broadcast after the filing of this suit shows that the district court's conclusion was not clearly erroneous. Sec 504(c)(1) also provides that statutory damages are available for all infringements involved in the suit wrt a single work, for which one infringer is liable, or two or more infringers are jointly and severally liable. Where more than one tortfeasor is jointly and severally liable, each work infringed is the basis for only one award, despite the number of infringements of the particular work. If ther are separate infringements for which the defs are not jointly liable, but are joined in one suit, than separate awards are proper. The

Chapter 11. Remedies, Pre-Emption and Related Bodies of Law 54/55

dist ct found that the TV stations involved here were not joint tortfeasors, and that their braodcasting of episodes of "Who's The Boss?" constituted separate acts of infringement. Sec 504(c)(1) also provides that statutory damages may be awarded for all infringements in an action wrt a single work. For purposes of the section, all parts of a compilation or derivative work comprise one work. The dist ct properly held that each episode constituted a separate work for the purposes of awarding damages. Thus, the dist ct did not err wrt its award of damages.

Chapter 11. Remedies, Pre-Emption and Related Bodies of Law 55/55

Вам также может понравиться